Все схемы 1 задания егэ биология 2018: Задание 1 ЕГЭ по биологии 2022: теория и практика

Содержание

Задание 1 ЕГЭ по биологии 2022: теория и практика

За это задание ты можешь получить 1 балл. Уровень сложности: базовый.
Средний процент выполнения: 53.2%
Ответом к заданию 1 по биологии может быть цифра (число) или слово.

Что нужно знать, чтобы решить задание 1:

Схемы и пропущенные термины, которые встречаются на экзамене, могут быть по любому разделу биологии, который нужно знать для ЕГЭ:

  1. Общая биология
  2. Анатомия
  3. Генетика
  4. Зоология
  5. Ботаника
  6. Экология
  7. Эволюция
  8. Селекция
Задача 1

Рассмотрите таблицу «Методы селекции» и заполните пустую ячейку, вписав соответствующий термин.

МетодПрименение метода
ЭлектрофорезДвижение заряженных частиц в электрическом поле, создаваемом внешним источником
 Из яйцеклетки удаляют ядро и в неё пересаживают ядро соматической клетки генетически ценного организма, затем стимулируют дробление реконструированной зиготы электрошоком и трансплантируют эмбрион в матку любой самки того же вида
Решение

Клонирование – получение нескольких генетически идентичных организмов путём бесполого (в том числе вегетативного) размножения. С помощью клонирования можно получить несколько идентичных организмов (не путать с генетическим клонированием, это к генной инженерии).

Ответ: клонирование

Показать решение

Полный курс
Задача 2

Рассмотрите таблицу «Классификация мутаций» и заполните пустую ячейку, вписав соответствующий термин.

Тип мутацииПример
ГеннаяГемофилия
 Болезнь Дауна
Решение

Синдром Дауна – самая распространенная хромосомная патология. Она возникает, когда в результате случайной мутации в 21-й паре появляется еще одна хромосома. Поэтому эту болезнь еще называют трисомия по 21-й хромосоме. Изменение числа хромосом – это геномные мутации.

Ответ: геномная

Показать решение

Полный курс
Задача 3

Рассмотрите таблицу «Уровни организации живой материи» и заполните пустую ячейку, вписав соответствующий термин.

УровеньПроцесс
МолекулярныйПередача генетической информации
 Катаболизм
Решение

Катаболизм — расщепление сложных веществ на более простые, и распад тканей и клеток.

Ответ: клеточный

Показать решение

Полный курс
Задача 4

Рассмотрите таблицу «Учёные и их открытия» и заполните пустую ячейку, вписав соответствующий термин.

Учёный
Открытие
 Двойное оплодотворение у покрытосеменных растений
ГарвейКруги кровообращения
Решение

Двойное оплодотворение — половой процесс у покрытосеменных растений, при котором оплодотворяются как яйцеклетка, так и центральная клетка зародышевого мешка. Двойное оплодотворение открыл русский учёный С. Г. Навашин в 1898 на 2 видах растений — лилии и рябчике.

Ответ: навашин

Показать решение

Полный курс
Задача 5

Рассмотрите таблицу «Уровни организации живой материи» и заполните пустую ячейку, вписав соответствующий термин.

УровеньПримеры
МолекулярныйБелки́, нуклеиновые кислоты
 Щитовидная железа
Решение

Щитовидная железа – это орган эндокринной системы, поэтому это органный уровень организации.

Ответ: органный

Показать решение

Полный курс
Задача 6

Рассмотрите таблицу «Уровни организации живой материи» и заполните пустую ячейку, вписав соответствующий термин без знаков препинания.

Уровень
Процессы
БиосферныйКруговорот веществ и превращение энергии
 Элементарные эволюционные изменения
Решение

Организмы одного вида, объединенные общим местообитанием, формируют популяцию. Вид обычно состоит из множества популяций. Популяции имеют общий генофонд. В пределах вида они могут обмениваться генами, т. е. являются генетически открытыми системами. В популяциях происходят элементарные эволюционные явления, приводящие в конечном итоге к видообразованию. Живая природа может эволюционировать только в надорганизменных уровнях.

Уровни организации жизни:

  • Молекулярный – молекулы белков, жиров, углеводов
  • Клеточный – клетки со всем комплексом органелл, возникает жизнь (на субклеточном уровне – отдельные органеллы). У бактерий, простейших, грибов и водорослей клеточный уровень соответствует организменному
  • Тканевый – (часто объединяют в тканево-органный уровень) – совокупность клеток (одинаковых по строению (органеллы)), происхождению (из какого зародышевого листка), функциям (что умеет: сокращаться или проводить импульсы, например) и межклеточного вещества.
  • Органный – органы – обособленные совокупности различных типов клеток и тканей, выполняющие определённую функцию в живом организме.
  • Организменный – отдельная особь (одноклеточный или многоклеточный организм)
  • Популяционно-видовой – виды и популяции. Вид крупнее популяции (!), он состоит из популяций. Популяция – совокупность особей одного вида, длительно проживающих на определенной территории, скрещивающихся друг с другом (при этом эта группа изолирована от других таких групп). Вид – совокупность особей, сходных по строению, имеющих общее происхождение, свободно скрещивающихся между собой и дающих плодовитое потомство.
  • Биогеоценотический – несколько разных видов + одна территория + неживая природа (состав воздуха, камни, почва) = экосистема/биогеоценоз
  • Биосферный – совокупность отношений между всеми организмами, обитающими на Земле
  • Ответ: популяционновидовой

    Показать решение

    Полный курс
    Задача 7

    Рассмотрите таблицу «Методы генетических исследований» и заполните пустую ячейку, вписав соответствующий термин.

    МетодПрименение метода
     Синдром Дауна
    ПопуляционныйЧастоты аллелей и генотипов
    Решение

    Синдром Дауна – патология, вызванная нарушением хромосомного набора, получаемого ребенком от родителей. У людей с этой патологией имеется лишняя 47-я хромосома, а в норме их должно быть 46. Число хромосом определяется цитогенетическим методом.

    Ответ: цитогенетический

    Показать решение

    Полный курс
    Задача 8

    Рассмотрите таблицу «Основные свойства живых систем» и заполните пустую ячейку, вписав соответствующий термин.

    СвойствоХарактеристика
    СаморегуляцияСпособность живых организмов поддерживать постоянство физико-химического состава, интенсивность физиологических процессов в меняющихся условиях окружающей среды
     Периодические изменения интенсивности физиологических процессов и функций с различными периодами колебаний
    Решение

    Свойства (признаки) живого:
    1. Единство химического состава – все живое на 98% из 4 биогенных элементов: углерод (С), кислород (О), азот (N) и водород (Н)
    2. Обмен веществ и энергии (метаболизм) – совокупность процессов поступления, превращения, использования, накопления и удаления продуктов распада веществ.
    3. Самовоспроизведение (репродукция) – способность воспроизводить себе подобных (репликация ДНК, деление клеток, бесполое и половое размножение, даже деление органоидов – митохондрий и пластид)
    4. Наследственность – способность организма передавать свои признаки из поколения в поколение
    5. Изменчивость – приобретение организмом (дочерним) новых признаков и свойств, отличных от признаков родительских форм
    6. Раздражимость – способность организма воспринимать раздражение из внешней и внутренней среды и избирательно реагировать на него (рефлексы и таксисы у животных; тропизмы, таксисы и настии у растений)

    7. Рост и развитие –
    Рост – количественное изменение — увеличением размеров или массы за счет увеличения размеров и количества клеток (за счет веществ, поступивших в процессе питания)
    Развитие – направленный необратимый процесс качественных (новый уровень с другими возможностями) изменений организма (его состав или структура: это может быть как усложнение, так и упрощение)
    Различают:
    • Онтогенез — индивидуальное развитие организма – последовательное проявление индивидуальных свойств организма и осуществление его роста в результате реализации генетического материала и под влиянием условий окружающей среды
    • Филогенез — историческое развитие — образование новых видов и прогрессивное усложнение жизни (эволюция)
    8. Ритмичность – периодические изменения интенсивности физиологических процессов через определенные равные промежутки времени
    9. Саморегуляция – способность организмов поддерживать постоянство внутренней среды (гомеостаз – постоянство химического состава и интенсивности протекания биологических процессов) в непрерывно меняющихся условиях внешней среды
    10. Энергозависимость – живые организмы открытые (поступает энергия извне) и динамические (устойчивые лишь при условии непрерывного доступа веществ и энергии: умрём без еды, воды, воздуха) системы
    11. Дискретность (иерархичность/целостность) – взаимная соподчиненность элементов и частей

    Ответ: ритмичность

    Показать решение

    Полный курс
    Задача 9

    Рассмотрите таблицу «Основные свойства живых систем» и заполните пустую ячейку, вписав соответствующий термин.

    СвойствоХарактеристика
    РостУвеличение размеров, массы и объёма организма
     Необратимое направленное закономерное изменение живых и неживых систем, в результате которого появляются качественно новые состояния систем
    Решение

    Свойства (признаки) живого:
    1. Единство химического состава – все живое на 98% из 4 биогенных элементов: углерод (С), кислород (О), азот (N) и водород (Н)
    2. Обмен веществ и энергии (метаболизм) – совокупность процессов поступления, превращения, использования, накопления и удаления продуктов распада веществ.
    3. Самовоспроизведение (репродукция) – способность воспроизводить себе подобных (репликация ДНК, деление клеток, бесполое и половое размножение, даже деление органоидов – митохондрий и пластид)
    4. Наследственность – способность организма передавать свои признаки из поколения в поколение
    5. Изменчивость – приобретение организмом (дочерним) новых признаков и свойств, отличных от признаков родительских форм
    6. Раздражимость – способность организма воспринимать раздражение из внешней и внутренней среды и избирательно реагировать на него (рефлексы и таксисы у животных; тропизмы, таксисы и настии у растений)
    7. Рост и развитие –
    Рост – количественное изменение — увеличением размеров или массы за счет увеличения размеров и количества клеток (за счет веществ, поступивших в процессе питания)
    Развитие – направленный необратимый процесс качественных (новый уровень с другими возможностями) изменений организма (его состав или структура: это может быть как усложнение, так и упрощение)
    Различают:
    • Онтогенез — индивидуальное развитие организма – последовательное проявление индивидуальных свойств организма и осуществление его роста в результате реализации генетического материала и под влиянием условий окружающей среды
    • Филогенез — историческое развитие — образование новых видов и прогрессивное усложнение жизни (эволюция)
    8. Ритмичность – периодические изменения интенсивности физиологических процессов через определенные равные промежутки времени
    9. Саморегуляция – способность организмов поддерживать постоянство внутренней среды (гомеостаз – постоянство химического состава и интенсивности протекания биологических процессов) в непрерывно меняющихся условиях внешней среды
    10. Энергозависимость – живые организмы открытые (поступает энергия извне) и динамические (устойчивые лишь при условии непрерывного доступа веществ и энергии: умрём без еды, воды, воздуха) системы
    11. Дискретность (иерархичность/целостность) – взаимная соподчиненность элементов и частей

    Ответ: развитие

    Показать решение

    Полный курс
    Задача 10

    Рассмотрите таблицу «Биология как наука» и заполните пустую ячейку, вписав соответствующий термин.

    Раздел биологииОбъект изучения
    АльгологияВодоросли
     Мхи
    Решение

    Бриология — один из разделов ботаники, занимающийся изучением мохообразных.

    Ответ: бриология

    Показать решение

    Полный курс
    Задача 11

    Рассмотрите таблицу «Методы биологических исследований» и заполните пустую ячейку, вписав соответствующий термин.

    МетодПрименение метода
    НаблюдениеСроки впадения в спячку сурков
     Влияние длины дня на цветение растений
    Решение

    Эксперимент — метод исследования некоторого явления в управляемых наблюдателем условиях. Отличается от наблюдения активным взаимодействием с изучаемым объектом. В данном случае исследователь изменяет длину дня, поэтому это пример эксперимента.

    Ответ: эксперимент

    Показать решение

    Полный курс
    Задача 12

    Рассмотрите таблицу «Учёные и сформулированные ими законы» и заполните пустую ячейку, вписав соответствующий термин. В ответе укажите только фамилию.

    УчёныйЗакон
    Ю. ЛибихЗакон минимума
     Закон сцепленного наследования признаков
    Решение

    В начале ХХ века Томас Морган, опираясь на данные исследователей, сформулировал основные положения хромосомной теории наследственности и установил закон сцепленного наследования признаков.

    Ответ: морган

    Показать решение

    Полный курс
    Задача 13

    Рассмотрите таблицу «Уровни организации живой материи» и заполните пустую ячейку, вписав соответствующий термин.

    УровеньПример
    ОрганизменныйСпирохета
     Хлоропласты
    Решение

    Хлоропласты – это органоиды клетки, которые участвуют в процессе фотосинтеза. Поэтому данный объект относится к субклеточному уровню организации.

    Ответ: субклеточный

    Показать решение

    Полный курс
    Задача 14

    Рассмотрите таблицу «Уровни организации живой материи» и заполните пустую ячейку, вписав в поле ответа, соответствующий термин.

    УровеньПроцессы
     Транскрипция, репликация
    ОрганизменныйГомеостаз, размножение
    Решение

    Транскрипция – это один из этапов синтеза белка, Репликация – это удвоение ДНК. То есть оба процесса происходят на молекулярном уровне. Поэтому данные процессы относятся к молекулярному уровню организации.

    Ответ: молекулярный

    Показать решение

    Полный курс
    Задача 15

    Рассмотрите таблицу «Учёные и их открытия» и заполните пустую ячейку, вписав в поле ответа, соответствующий термин.

    УчёныйОткрытие
    Р. БроунКлеточное ядро
     Сперматозоиды человека
    Решение

    Первым, кто описал мужскую гамету, был голландский ученый Антони ван Левенгук. Открытие датируется 1677 годом. Как сообщал в своих трактатах сам микроскопист, поведал о сперматозоидах ему друг Иоганн Гамм. Формально он и является первооткрывателем, но именно рассмотрел и сделал зарисовки в деталях сперматозоиды, именно Левенгук.

    Ответ: левенгук

    Показать решение

    Полный курс
    Задача 16

    Рассмотрите таблицу «Уровни организации живой материи» и заполните пустую ячейку, вписав соответствующий термин. Если организм находится на двух уровнях жизни, в ответе укажите оба без пробелов, начиная с наименьшего.

    УровеньПример
     Инфузория-туфелька
    Популяционно-видовойТабун диких лошадей
    Решение

    Инфузория-туфелька – это одноклеточный организм. Клетка выполняет функции целого организма. Поэтому данный организм находится как на клеточном (в связи с особенностями строения) уровне, так и на организменном.

    Ответ: организменныйклеточный

    Показать решение

    Полный курс
    Задача 17

    Рассмотрите таблицу «Биология как наука» и заполните пустую ячейку, вписав в поле ответа, соответствующий термин.

    Раздел биологииОбъект изучения
    ГенетикаНаследственность и изменчивость
     Гомеостаз
    Решение

    Физиология — биологическая наука, изучающая жизнедеятельность здорового организма и его частей. Т.е. физиология изучает все процессы, протекающие в организме, в частности гомеостаз (механизм постоянства состава внутренней среды).

    Ответ: физиология

    Показать решение

    Полный курс
    Задача 18

    Рассмотрите таблицу «Уровни организации живой материи» и заполните пустую ячейку, вписав в поле ответа, соответствующий термин.

    УровеньНауки, которые изучают живые организмы на этом уровне
     Гистология
    Популяционно-видовойГенетика, эволюция, экология
    Решение

    Гистология – наука о тканях, поэтому она изучает тканевый уровень организации.

    Ответ: тканевый

    Показать решение

    Полный курс
    Задача 19

    Рассмотрите таблицу «Методы биологических исследований» и заполните пустую ячейку, вписав в поле ответа, соответствующий термин.

    МетодПрименение метода
     Метод, основанный на разной скорости движения через адсорбент растворённых в специальном растворе веществ; при пропускании такого раствора через адсорбент каждое вещество из смеси передвигается на определённое расстояние в зависимости от своей молекулярной массы (в качестве адсорбента используют волокна фильтровальной бумаги, порошок целлюлозы и др.)
    ЦентрифугированиеРазделение частей клеток, отличающихся по удельному весу, с помощью центрифуги; выделение разных компонентов клетки и их исследование
    Решение

    Хроматография — динамический сорбционный метод разделения и анализа смесей веществ, а также изучения физико-химических свойств веществ. Основан на распределении веществ между двумя фазами — неподвижной (твердая фаза или жидкость, связанная на инертном носителе) и подвижной (газовая или жидкая фаза, элюент). Название метода связано с первыми экспериментами по хроматографии, в ходе которых разработчик метода Михаил Цвет разделял ярко окрашенные растительные пигменты. Метод хроматографии был впервые применён русским учёным-ботаником Михаилом Семеновичем Цветом в 1900 году.

    Ответ: хроматография

    Показать решение

    Полный курс
    Задача 20

    Рассмотрите таблицу «Биология как наука» и заполните пустую ячейку, вписав в поле ответа, соответствующий термин.

    Раздел биологииОбъект изучения
    микробиологияпрокариоты
     грибы
    Решение

    Микробиология – это наука, изучающая организмы, не видимые невооруженным взглядом, в частности прокариоты (организмы не имеющие ядра). Микология – наука изучающая грибы.

    Ответ: микология

    Показать решение

    Полный курс

    Советы по оформлению и решению задания №1:

    1. Обращай внимание на правильность написания термина, потому что твоя тестовая часть будет проверяться компьютером. Написав слово «митАхондрия», вместо «митОхондрия», ты потеряешь балл.
    2. Вноси свой ответ в бланк без знаков препинания, тире и прочих символов.
    3. Не торопись и не паникуй при решении задания.
    4. Если не можешь вспомнить термин или сомневаешься в себе, то переходи к выполнению остальных задач, а потом возвращайся и подумай еще.

    Примеры задания из разных разделов биологии.

    Пример 1. Рассмотрите предложенную схему «Типы развития насекомых». Запишите в ответе пропущенный термин, обозначенный на схеме знаком вопроса.

    Для решения задания нужно знать этапы постэмбрионального развития насекомых.

    Для насекомых характерно непрямое развитие. Что это значит? — Новая особь рождается непохожей на взрослый организм и нуждается в превращении (метаморфозе). Метаморфоз может быть полным и неполным.

    Неполное превращение (неполный метаморфоз). Самка откладывает яйца в окружающую среду, из которых вылупляется личинка. Личинка линяет и постепенно превращается во взрослую особь (имаго).
    3 стадии развития: яйцо – личинка – взрослый организм (имаго).

    Полное превращение (полный метаморфоз). Самка откладывает яйца, из которых появляется личинка. Личинка растет и превращается в куколку. Внутри куколки развиваются абсолютно все органы и формируется взрослый вид насекомого (имаго).
    4 стадии: яйцо – личинка – куколка — взрослый организм (имаго).

    Ответ: куколка

    Пример 2. Рассмотрите предложенную схему «Форменные элементы крови». Запишите в ответе пропущенный термин, обозначенный на схеме знаком вопроса.

    Для решения задания нужно знать классификацию лейкоцитов.

    Кровь состоит из плазмы и форменных элементов. К форменным элементам (клеткам крови) относят лейкоциты, эритроциты и тромбоциты. Лейкоциты классифицируются на: гранулоциты (нейтрофилы, базофилы, эозинофилы) и агранулоциты (моноциты и лимфоциты).

    Ответ: лимфоциты

    Еще больше крутых лайфхаков, разборов, ловушек ЕГЭ и теории в нашей группе вконтакте и инсте преподавателей @turboegebio и @turbobio

    Критерии оценивания заданий егэ по биологии.

    В 2017 году ЕГЭ по биологии ждут большие изменения. Предстоящая реформа проводится в соответствии с планами министерства образования, обнародованных еще несколько лет назад. Цель реформирования – уйти от привычной тестовой системы, которая, по словам самих чиновников, дает достаточно ненадежные результаты. И в 2017 году биология стала очередным предметом, из которого убирают все вопросы-тесты.

    Дата проведения

    • 03.2017г. – первый этап, досрочный;
    • 04.2017г. – резервный день первого этапа;
    • 06.2017г. – второй этап, основной;
    • 06.2017г. – резервный день второго этапа;
    • 06.2017г. – резервный день по всем предметам.

    Изменения в ЕГЭ по биологии в 2017 году

    В 2017 году школьников ждут реформы сразу по трем предметам: химии, физике, биологии. Какие изменения внесены в ЕГЭ по биологии? В первую очередь это удаление тестовой части и замена ее на блоки с краткими ответами.

    • Изменение количества заданий. В ЕГЭ 2017 будет 28 заданий, а не 40, как это было раньше.
    • Введение заданий новых типов. Вместо заданий с выбором одного правильного ответа из четырех предложенных вариантов в экзамен вошли задачи на дополнение или восстановление таблиц и схем. Проверка навыков работы с графическим материалом – рисунками, схемами, таблицами, графиками – становится важной частью экзамена по биологии.
    • Увеличение продолжительности экзамена. Время, отведенное на ЕГЭ по биологии 2017, увеличится до 210 минус (против 180 в прошлом году).
    • Уменьшение проходного балла. Первичный/пороговый балл по биологии в 2017 году снижен до 59 (против 61 в прошлом году).

    Структура экзамена:

    • Рассмотрение данных в таблице или графике – 1 задание
    • Заполнение таблиц — 1 задание
    • Заполнение схем – 1 задание
    • Решение задач по генетике/цитологии – 2 задания
    • Определение последовательности в процессах/явлениях – 3 задание
    • Поиск соответствий – 6 заданий

    Примерная структура экзамена соблюдена во всех базах и демонстрационных версиях ЕГЭ, а что существенно облегчает подготовку. Особо стоит отметить, что новая модель КИМ сходна с вариантом ОГЭ по биологии, сдаваемом в девятом классе. Так что будущие выпускники могут не беспокоиться – структура экзамена не окажется чем-то принципиально новым для них.

    Оценка за ЕГЭ

    Минимальное количество баллов, которое необходимо набрать на биологии – 36, а первичный балл равняется 59. За правильные ответы на некоторые вопросы ученик может получить один балл, а за некоторые — три.

    • Вопросы 1, 3, 6 – ответы на них оцениваются в 1 балл;
    • Вопросы 2, 4, 5, 7, 8, 9, 10, 11, 12, 13, 14, 15, 16, 17, 18, 19, 20, 21, 22 – за правильные ответы можно получить 2 балла;
    • Вопросы 23, 24, 25, 26, 27, 28 – правильные ответы приносят 3 балла.

    По предупреждениям ФИПИ, некоторые вопросы осознанно усложнены. Ответы на них по силам лишь тем, кто знает биологию на «отлично». Такие вопросы потребуют не просто краткого ответа, а приведения аргументов в пользу выбранной версии и подробного пояснения хода мыслей.

    Как готовиться к экзамену

    Чтобы помочь школьникам легко и беспроблемно сдать ЕГЭ по биологии, ФИПИ публикует вспомогательные и методические материалы.

    Подготовка к ЕГЭ по биологии 2017 должна обязательно включать в себя работу с демоверсиями экзамена. На сегодняшний день существует 36 вар вариантов КИМов, причем все они со сходной структурой, то есть для подготовки можно взять любой из предложенных ФИПИ вариантов.

    Методисты уверяют – подготовка к экзамену займет довольно продолжительное время, и для успешного освоения материала лучше всего составить график и придерживаться его. В подготовке школьникам наверняка помогут созданные специалистами ФИПИ рекомендации:

    1. Ориентируясь на опубликованные ФИПИ темы, составьте список подразделов, которые необходимо повторить. Исходя из объема и сложности темы, составьте график их изучения, при этом на сложные темы необходимо отвести больше времени, чем на простые и легкие.
    2. Изучать материал лучше всего по школьному учебнику. В качестве дополнительных источников можно использовать рекомендованные ведомством методические материалы и информацию из всемирной сети.
    3. Чтобы облегчить изучение темы и облегчить запоминание материала, сведите данные по изучаемой теме в таблицу или схематичный конспект.
    4. Используйте опубликованные на сайте ФИПИ КИМы для подготовки. Благодаря этому вы сможете обнаружить пробелы в знаниях по определенным темам.
    5. Изучая новые темы, периодически возвращайтесь к старым, повторяя и освежая в памяти пройденный материал.

    Демоверсии экзамена помогут в подготовке к экзаменам. Помимо уже упомянутой возможности обнаружить слабо изученные темы, онлайн-версии помогают познакомиться с самой структурой экзамена.

    Вывод

    Рассматривая все внесенные изменения, приходим к малоутешительному (для части школьников) выводу – ЕГЭ по биологии в 2017 году будет сдать значительно сложнее, чем в 2016. Сложнее будет получить как минимальное количество баллов, то есть достичь порога проходимости, так и набрать максимальное значение. Пропадает шанс получить баллы за угадывание, придется действительно готовиться к экзамену и досконально изучать предмет.

    ДатаЕГЭ
    Досрочный период
    20 марта (ср)география, литература
    22 марта (пт)русский язык
    25 марта (пн)история, химия
    27 марта (ср)иностранные языки (устно)
    29 марта (пт)математика Б, П
    1 апреля (пн)иностранные языки, биология, физика
    3 апреля (ср)обществознание, информатика и ИКТ
    5 апреля (пт)резерв: география, химия, информатика и ИКТ, иностранные языки (устно), история
    8 апреля (пн)резерв: иностранные языки, литература, физика, обществознание, биология
    10 апреля (ср)резерв: русский язык, математика Б, П
    Основной этап
    27 мая (пн)география, литература
    29 мая (ср)математика Б, П
    31 мая (пт)история, химия
    3 июня (пн)русский язык
    5 июня (ср)иностранные языки (письменно), физика
    7 июня (пт)иностранные языки (устно)
    8 июня (сб)иностранные языки (устно)
    10 июня (пн)обществознание
    13 июня (чт)биология, информатика и ИКТ
    17 июня (пн)Резерв: география, литература
    18 июня (вт)Резерв: история, физика
    20 июня (чт)Резерв: биология, информатика и ИКТ, химия
    24 июня (пн)Резерв: математика Б, П
    26 июня (ср)Резерв: русский язык
    27 июня (чт)Резерв: иностранные языки (устно)
    28 июня (пт)Резерв: обществознание, иностранные языки (письменно)
    1 июля (пн)Резерв: по всем учебным предметам

    В 2018 г. в ЕГЭ по биологии приняли участие более 133 тыс. человек, что несколько больше, чем число участников экзамена в 2017 г. (111 748) человек, 2016 г. (126 006) и 2015 г. (122 936). Экзамен по биологии традиционно востребован и входит в пятерку самых популярных выпускных экзаменов по выбору. Его выбирают мотивированные на биологию выпускники, поступающие в медицинские, ветеринарные, аграрные вузы, психологические и биологические факультеты университетов, академий и институтов физической культуры и спорта и ряд других вузов.

    В 2018 г. средний тестовый балл составил 51,4. Его незначительное снижение по сравнению с прошлым годом связано в первую очередь с уменьшением доли участников в интервале тестового балла 61–80 (на 2,26%) и увеличением доли участников в интервале 41–60 (на 3,26%). При этом отмечается стойкая тенденция к снижению числа участников с результатами в интервале 81–100. Сокращение доли высокобалльников можно объяснить комплексом мер по повышению качества проверки развернутых ответов экспертами предметных комиссий регионов, а также включением в часть 2 КИМ новых сюжетов конкретного, контекстного, практико-ориентированного характера, требовавших четкой аргументации, а не воспроизведения общих или частных знаний по предмету. Это позволило провести более качественную дифференциацию участников с высокими тестовыми баллами.

    Минимальный тестовый балл в 2018 г., как и в предыдущие годы, составил 36 баллов, а первичный — 16 баллов. Доля участников ЕГЭ по биологии, не набравших минимального количества баллов в 2018 г., составила 17,4%. По сравнению с 2017 г. доля участников, набравших тестовые баллы в диапазоне 41–60, составила 40,6% (в 2017 г. – 37,3%), а в диапазоне 61–80 составила 25,6% (в 2017 – 27,9%).

    В 2018 г. выполнили все задания экзаменационной работы и набрали 100 баллов 48 выпускников, что составило 0,04% от общего числа участников ЕГЭ. Полученные данные свидетельствует, с одной стороны, о достаточно высоком уровне сложности модели КИМ ЕГЭ 2018 г. и, с другой стороны, о доступности заданий, что подтверждается распределением первичных и тестовых баллов участников.

    Более подробные аналитические и методические материалы ЕГЭ 2018 года доступны по ссылке .

    На нашем сайте представлены около 5500 заданий для подготовки к ЕГЭ по биологии в 2018 году. Общий план экзаменационной работы представлен ниже.

    ПЛАН ЭКЗАМЕНАЦИОННОЙ РАБОТЫ ЕГЭ ПО БИОЛОГИИ 2019 ГОДА

    Обозначение уровня сложности задания: Б — базовый, П — повышенный, В — высокий.

    Проверяемые элементы содержания и виды деятельности

    Уровень сложности задания

    Максимальный балл за выполнение задания

    Задание 1. Биологические термины и понятия. Дополнение схемы
    Задание 2. Биология как наука. Методы научного познания. Уровни организации живого. Работа с таблицей
    Задание 3. Генетическая информация в клетке. Хромосомный набор, соматические и половые клетки. Решение биологической задачи
    Задание 4. Клетка как биологическая система. Жизненный цикл клетки. Множественный выбор (с рис. и без рис.)
    Задание 5. Клетка как биологическая система. Строение клетки, метаболизм. Жизненный цикл клетки. Установление соответствия (с рис. и без рис.)
    Задание 6. Моно- и дигибридное, анализирующее скрещивание. Решение биологической задачи
    Задание 7. Организм как биологическая система. Селекция. Биотехнология. Множественный выбор (без рис. и с рис.)
    Задание 8. Организм как биологическая система. Селекция. Биотехнология. Установление соответствия (с рис. и без рис.)
    Задание 9. Многообразие организмов. Бактерии, Грибы, Растения, Животные, Вирусы. Множественный выбор (с рис. и без рис.)
    Задание 10. Многообразие организмов. Бактерии, Грибы, Растения, Животные, Вирусы. Установление соответствия (с рис. и без рис.)
    Задание 11. Многообразие организмов. Основные систематические категории, их соподчиненность. Установление последовательности
    Задание 12. Организм человека. Гигиена человека. Множественный выбор (с рис. и без рис.)
    Задание 13. Организм человека. Установление соответствия (с рис. и без рис.)
    Задание 14. Организм человека. Установление последовательности
    Задание 15. Эволюция живой природы. Множественный выбор (работа с текстом)
    Задание 16. Эволюция живой природы. Происхождение человека. Установление соответствия (без рис.)
    Задание 17. Экосистемы и присущие им закономерности. Биосфера. Множественный выбор (без рис.)
    Задание 18. Экосистемы и присущие им закономерности. Биосфера. Установление соответствия (без рис.)
    Задание 19. Общебиологические закономерности. Установление последовательности
    Задание 20. Общебиологические закономерности. Человек и его здоровье. Работа с таблицей (с рис. и без рис.)
    Задание 21. Биологические системы и их закономерности. Анализ данных, в табличной или графической форме
    Задание 22 (С1). Применение биологических знаний в практических ситуациях (практико-ориентированное задание)
    Задание 23 (С2). Задание с изображением биологического объекта
    Задание 24 (С3). Задание на анализ биологической информации
    Задание 25 (С4). Обобщение и применение знаний о человеке и многообразии организмов.
    Задание 26 (С5). Обобщение и применение знаний в новой ситуации об эволюции органического мира и экологических закономерностях в новой ситуации
    Задание 27 (С6). Решение задач по цитологии на применение знаний в новой ситуации.
    Задание 28 (С7). Решение задач по генетике на применение знаний в новой ситуации

    Соответствие между минимальными первичными баллами и минимальными тестовыми баллами 2018 года. Распоряжение о внесении изменений в приложение № 2 к распоряжению Федеральной службы по надзору в сфере образования и науки.

    ЕГЭ по биологии 2018 – экзамен по выбору, он совершенно необязателен. В то же время данный предмет занимает 5 место после обязательных русского языка и математики, затем идут обществознание и физика. 5 место – за биологией (её выбирают примерно 18% выпускников).

    Где пригодится биология?

    Биологию сдают во многих вузах:

    1. медицинском;
    2. вузах биологического профиля;
    3. в педагогическом на специальность «Учитель биологии»;
    4. в сельскохозяйственном;
    5. ветеринарном;
    6. на физкультурном факультете;
    7. психологическом;
    8. экологическом;
    9. на факультете садового дизайна;
    10. на биологическом факультете в технических вузах, где биология изучается на стыке с физикой.

    Профессий, связанных с биологией очень много:

    • инженер, который моделирует проблемы, связанные с жизнью человека;
    • врач, который займётся лечением болезней, возможных в будущем;
    • эколог, заботящийся о здоровье всей страны. Задача эколога – создать условия, в которых человечество будет пить чистую воду, дышать чистым воздухом;
    • психолог;
    • спортсмен.

    Это те люди, которые делают жизнь человека более полноценной и значимой и, более того, продлевают эту жизнь.

    Необходимые документы

    На сайте ФИПИ размещены следующие документы, нужные для подготовки к ЕГЭ по истории:

    1. Спецификация (описание работы, перечислены необходимые документы, отражена структура ЕГЭ по биологии, дан план варианта КИМа).
    2. Кодификатор (перечень умений и тем, которые проверяются на ЕГЭ по биологии).
    3. Демоверсия ЕГЭ по биологии (один вариант ЕГЭ по биологии), с которой и нужно начинать подготовку к государственному экзамену по истории.

    Особенности структуры ЕГЭ по биологии

    Всего 28 заданий
    1 часть 2 часть
    21 задание с кратким ответом 7 заданий с развёрнутым ответом

    Время на выполнение всей работы – 3 часа 30 минут (210 минут).
    Максимальный первичный балл – 59.
    Установленный минимальный тестовый балл на ЕГЭ по биологии составляет 36 баллов.

    Как устроены ключевые задания 1 части работы?

    Задание №1 впервые появилось в 2017 году. Каждый ученик, готовясь к экзамену, читает текст, подчёркивает ключевые слова, затем находит ключевые слова и связи между ними. Эти связи дают возможность выстроить зависимость. Таким образом, текст структурируется, в нем находятся связи между понятиями. В данном задании представлен некий фрагмент из биологии, где и требуется показать эти связи.

    Задание №3 интересно тем, что введены обычные простые задачи, где требуется выполнить решение, зная биологию:

    • посчитать число хромосом,
    • указать количество клеток, которые образуются при различных процессах и т. д.

    Задание №4 требует подобрать два утверждения, которые соответствуют описанию того объекта, который представлен.
    Задание №6 на генетику. Это задачи на моногибридное скрещивание, где требуется написать конкретное число, соотношение цифр.
    Задание №8 на соотношение примеров и явлений. Это первый пример задания. Предлагается и альтернативный пример этого задания, где даётся слепой рисунок (без подписей), и нужно найти позиции, где есть цифры 1 и 2. И только после этого, определившись с позицией, подбирать правильные ответы.
    В задании №9 следует найти из предложенного текста примеры, которые соответствуют понятию «устойчивость», «микроскопичность» и «патогенность».
    Задание №15 на знание видовых свойств. В биологии понятие «вид» – это ключевое понятие, вокруг которого строится вся биология. Дан текст. Надо отобрать только те критерии, которые соответствуют поставленной в тексте задаче.

    Задание №20. Новое (с 2017 года), оригинальное задание, дающее возможность на одном примере проверить сразу несколько очень важных, ключевых понятий в биологии.

    • Пример 1 проверяет понимание видообразования.

    Пример 2 предполагает заполнение свободных граф в таблице и нахождение неких взаимосвязей между строением, объектом и функцией.

    Задание №21.

    • Пример 1. Формирование естественно-научной грамотности и исследовательских умений возможно только тогда, когда человек умеет работать с информацией, представленной в различных видах. В биологии, например, такой информацией является таблица, где учёный, ведя наблюдение, фиксирует данные, записывает их, проводя некий расчёт. Абитуриенту надо прочитать, что здесь отмечено, как это читает биолог, который из частокола цифр выбирает ключевые и на этом основании строит умозаключения. Надо выбрать только 2 утверждения, которые строго соответствуют заданным условиям.
    • Пример 2 требует выбрать утверждения, которые можно сформулировать на основании анализа гистограммы. Например, надо найти виды, посмотреть, чем они питаются, зашифровать этот вид (как бы обезличить) и предложить соотношение пищевых пристрастий этого вида. А потом необходимо оценить этот вид, исходя из его пищевых пристрастий или каких-то других качеств.
    • Пример 3 – это графическое представление. Здесь проверяется умение читать график и понимать причины движения кривой вверх или вниз.

    Несколько слов о части 2

    Это задания в открытой форме, где выпускнику придётся отвечать на вопрос и демонстрировать глубокие знания предмета. Особое внимание надо уделить языку изложения ответа. Ученик 11 класса должен владеть определённым понятийным аппаратом, биологическими терминами, используемыми в ответах к заданиям.

    Что поможет эффективно помочь сдать ЕГЭ по биологии?

    Конечно же, учебники, которыми пользуются в школах. Это могут быть учебники разных авторов, которые можно комбинировать.

    Чёткое планирование своей деятельности по подготовке к ЕГЭ по биологии 2018.

    Глубокая заинтересованность ученика в изучении и сдаче предмета на высокий балл.

    Надо быть мотивированным, надо хотеть изучить предмет биологию, и тогда результат не заставит себя ждать.

    Дата проведения ЕГЭ по биологии будет известна в январе 2018 года.

    О результатах ЕГЭ по биологии в 2018 году можно узнать в своей общеобразовательной организации или на официальном сайте ЕГЭ.

    Если Министерство образования решит внести какие-то изменения в КИМы ЕГЭ по биологии, то надо быть ко всему готовыми, не терять времени даром и начинать готовиться уже сейчас!

    Профессий, связанных с биологией, очень много. И если выпускник решил дальнейшую свою жизнь посвятить медицине, психологии, педагогике, технологии пищевой промышленности, фармакологии или сельскому хозяйству, то экзамен по биологии по окончании общеобразовательной школы для него обязателен. Каким будет ЕГЭ по биологии 2017 года ?

    Государственный экзамен по биологии в 2017 году будет отличаться от экзаменов предыдущих лет. Изменений внесено не много, но они существенные.

    Предлагается новая, более оптимизированная структура экзаменационной работы. Сокращено количество заданий до 28, по сравнению с 40 в 2016 году. Уменьшено максимальное количество первичных балов за всю работу, теперь это 59 баллов, вместо 60 в прошлом году. А вот количество времени для выполнения экзаменационной работы увеличено до 210 минут, вместо 180.

    В экзамене теперь не будет вопросов с выбором одного правильного ответа. В связи с этим вероятность угадывания сведена к минимуму. Ведь раньше была возможность подключить интуицию, пройти порог и набрать минимально допустимое количество балов. Теперь интуиция нам не помощник, нужны знания. Готовиться придется тщательно, и начинать как можно раньше. Но есть и хорошая новость. Сложность заданий не изменилась, а время на их выполнение увеличилось на 30 минут.

    Экзаменационная работа ЕГЭ по биологии 2017 содержит 28 заданий, состоит из двух частей, каждая из которых имеет свой уровень сложности и формат.

    Первая часть экзамена

    Первая часть состоит из 21 задания с кратким ответом. Причем ответы нужно будет записывать в виде слов или словосочетаний, числа или последовательности чисел, без пробелов и разделительных знаков.

    В первую часть включены задания двух уровней сложности: 10 — базового уровня и 11- повышенного.

    Такой формат — первое новшество и весомая причина для прохождения демоверсии ЕГЭ по биологии 2017.

    Второе новшество в содержании заданий. Выпускнику нужно будет:

    • анализировать информацию в графической или табличной форме (1 задание)
    • дополнять недостающую информацию в схеме и таблице (2 задания)
    • устанавливать последовательность систематических таксонов, биологических объектов, явлений, процессов (3 задания)
    • решать биологические задачи по цитологии и генетике (2 задачи)
    • выполнять задания с множественным выбором (7 заданий) и устанавливать соответствия (6 заданий) с рисунком или без него

    Итак, 21 задание нового типа имеют существенные различия и позволяют точнее оценить знание пройденного материала, а также требуют серьезной подготовки.

    Вторая часть экзамена (повышенная сложность)

    Вторая часть ЕГЭ по биологии 2017 ориентирована на выпускников имеющих высокий уровень знаний по предмету, а точнее на их выявление. Здесь не будет никаких изменений. Как и в 2016 году, будет 7 заданий, сформированных по видам учебной деятельности и в соответствии с темами предмета. Ответы на них выпускники пишут самостоятельно в развернутой форме. По уровню сложности также изменений нет: 1 задание повышенного и 6 высокого уровня.

    Во второй части выпускник должен самостоятельно объяснять и обосновывать биологические явления и процессы, уметь проводить анализ, систематизацию и интеграцию знания, подтверждать теорию практикой. И все это грамотно формулировать в развернутом ответе.

    Как оценивается

    Первичное максимальное количество балов за государственный экзамен в 2017 году изменилось, но не существенно.

    За выполнение 10 заданий базового уровня сложности дается 17 баллов, за 12 заданий повышенного уровня — 24 балла, за 6 заданий высокого уровня — 18. В сумме — 59 баллов. для сдачи экзамена по биологии составит 36 баллов.

    Что нужно знать

    Экзамен предполагает проверку знания предмета в полном объеме. И если что-то упущено, даже по уважительной причине, на экзамене этот факт не принимается во внимание. Следовательно, будущему выпускнику желательно начинать подготовку к уже в сентябре, чтобы успеть восполнить все пробелы.

    Проверка знаний по биологии будет проводиться по следующим разделам:

    • Биология как наука. Методы научного познания
    • Клетка как биологическая система
    • Организм как биологическая система
    • Система и многообразие биологического мира
    • Организм человека и его здоровье
    • Эволюция живой природы
    • Экосистемы и присущие им закономерности

    Воспользуйтесь этим списком, чтобы проверить себя и подтянуть упущенные темы.

    Что нужно уметь

    На ЕГЭ по биологии также важно показать свои умения и навыки использования полученных знаний. Выпускник должен правильно применять терминологию, определять биологические объекты не только по описанию, но и по рисунку. При объяснении биологических процессов существенное преимущество будут иметь те, кто использует не только слово, а и таблицы, графики, схемы. Решать задачи, делать выводы и применять теоретические знания на практике, в повседневной жизни.

    Каждый год ФИПИ выпускает обновленную демонстрационную версию ЕГЭ по биологии. У выпускника есть возможность спокойно разобраться с инструкциями, заданиями, правилами проведения, критериями оценивания, практически пройти все этапы экзамена. Безусловно, демоверсия это не точная копия ЕГЭ, но, тем не менее, она дает возможность ученику чувствовать себя спокойнее и увереннее, когда все будет происходить по-настоящему.

    В подготовке к ЕГЭ по биологии каждому выпускнику необходимо учесть свои личные, индивидуальные особенности. Каждый знает свои слабые и сильные стороны, возможности памяти, работоспособности. Кто в этом еще не разобрался, может обратиться за помощью к родителям и учителям. Стоит пообщаться с теми, кто уже сдавал ЕГЭ и получить дополнительную информацию из первых рук. Одним словом, для подготовки все способы хороши, главное не оставлять все на май, не надеяться на «авось пронесет», а начинать систематически заниматься уже сейчас.

    Биология относится к тем предметам, которые выпускник на ЕГЭ выбирает сам. И если он делает выбор в пользу биологии, значит эта наука ему по душе. Поэтому все тревоги и переживания нужно оставить за дверью, чтобы не мешали. А с собой взять собранность, уверенность, вспомнить все и получить высший балл.

    Видео новость, демоверсии

    ЕГЭ по биологии 2018. Задание 26

    Биология. ЕГЭ 2018 года. Особенности

    • С 2018 года будет уделяться особое внимание понятийному аппарату. Не исключено, что уже в будущем году в кодификаторах к экзамену пропишут терминологию, которой должен владеть учащийся.

    • Увеличится число вопросов, связанных с эрами и периодами, эволюцией растений и животных. Раньше они почти не встречались. Выпускникам необходимо изучить геохронологическую таблицу, быть готовым к заданиям на определение организма, относящегося к тому или иному периоду, к той или иной эре, на определение отпечатка, на расчет времени существования данного организма.

    • Увеличится число вопросов, посвященных происхождению жизни на Земле. Раньше их практически не было.

    • Почти исключится формулировка «Что произойдет, если исчезнут …». Некоторые последствия невозможно предугадать. Взамен абитуриентам будет предложено назвать факторы, которые могут привести к сокращению численности той или иной популяции.

    • В этом году абитуриентам не удастся ограничиться лаконичными ответами. Экзаменуемые должны показать знание предмета — для этого при подготовке необходимо уйти от «натаскивания».

    ЕГЭ. Биология. Новый полный справочник для подготовки к ЕГЭ

    Вниманию учащихся и учителей предлагается новое учебное пособие, которое поможет успешно подготовиться к единому государственному экзамену по биологии. Справочник содержит весь теоретический материал по курсу биологии, необходимый для сдачи ЕГЭ. Он включает в себя все элементы содержания, проверяемые контрольно-измерительными материалами, и помогает обобщить и систематизировать знания и умения за курс средней (полной) школы. Теоретический материал изложен в краткой, доступной форме. Каждый раздел сопровождается примерами тестовых заданий, позволяющими проверить свои знания и степень подготовленности к аттестационному экзамену. Практические задания соответствуют формату ЕГЭ. В конце пособия приводятся ответы к тестам, которые помогут школьникам и абитуриентам проверить себя и восполнить имеющиеся пробелы. Пособие адресовано школьникам, абитуриентам и учителям.

    Купить

    Варианты задания № 26, решения и пояснения

    Пример 1
    Какие ароморфозы обеспечили развитие древнейших организмов в архее и протерозое? Укажите не менее четырех ароморфных признаков и их значение в эволюции. Элементы ответа:
    Элементы ответа:
    1. Появление фотосинтеза обеспечило первичный синтез органических веществ из неорганических, накопление кислорода в воде и атмосфере, образование озонового экрана.

    2. Появление аэробного обмена веществ обеспечило синтез большого количества АТФ и снабжение организма энергией.

    3. Половой процесс привел к появлению у организмов разнообразных признаков — материала для эволюции.

    4. Появление многоклеточности привело к дифференциации клеток, тканей и органов.

    5. Появление эукариот обеспечило разнообразие царств живой природы.

    Примечание: Формулировки ответа могут быть разные, главное — правильное употребление терминов. Кроме того, если экзаменуемый не выполнит задание «Укажите», «Объясните», то потеряет 2 балла.


    Пример 2

    Популяция стабильна, если она имеет большую численность. Почему вероятность исчезновения малочисленных популяций выше, чем многочисленных?

    Элементы ответа:
    1. Вероятность гибели малочисленных популяций от воздействия неблагоприятных факторов внешней среды выше, чем у многочисленной популяции.

    2. Ослабевают связи между членами популяции (звуковая сигнализация, выделение химических веществ).

    3. Затруднена репродукция потомства.

    4. Близкородственное скрещивание особей в малочисленных популяциях приводит к появлению вредных рецессивных генов. При высокой численности популяция относительно гетерогенна.

    Примечание: В задании не указывается количество критериев, на которые нужно дать ответ, — следовательно, необходимо назвать не меньше трех. Ответ должен соответствовать смыслу вопроса, и если этого соответствия нет, то снижается балл. Для выполнения задания нужно указать события, которые ведут к исчезновению малых популяций. Критерий № 2 наиболее сложный для абитуриентов, он редко упоминается в алгоритме для решения подобной задачи.


    Пример 3

    У трески, щуки и многих других рыб количество выметываемых икринок исчисляется миллионами. Вместе с тем, имеются рыбы, которые мечут несколько сотен или десятков икринок. Объясните, почему в природе существуют те и другие рыбы.

    Элементы ответа:
    1. Самки рыб, как правило, выметывают большое количество икры в воду и она там оплодотворяется. Оплодотворение внешнее.

    2. Приспособленность к выживанию при внешнем оплодотворении — большое количество икры.

    3. У рыб с небольшой плодовитостью хорошо развита забота о потомстве, иначе они не могли бы существовать.

    Примечание: Подобные задания вызывают сложности. Критерий № 1 трудно вывести из вопроса, поскольку у подавляющего большинства рыб наружное оплодотворение. Если экзаменуемый ответит, что часть икры не оплодотворяется, выбрасывается волнами на берег или поедается другими рыбами, это будет в большей степени соответствовать смыслу вопроса. Нужно познакомить учеников с разными вариантами ответа.


    Пример 4

    Считается, что на склонах холмов поля надо распахивать поперек склона (горизонтально, террасами), а не вдоль (от вершины к подножию). Объясните, почему необходимо делать именно так и к чему может привести распашка полей вдоль склона.

    Элементы ответа:
    1. При распашке вдоль склона и вода, используемая для полива, и естественные осадки будут стекать вдоль грядок к подножию холма.

    2. Эта вода будет вымывать из почвы удобрения и другие полезные вещества, ускоряя эрозию почвы.

    3. При распашке поперек склона вода будет дольше оставаться в почве и вещества будут вымываться гораздо медленнее.

    Примечание: Школьники, как правило, не знакомы с термином «террасирование». Это задание вызывает у них серьезные затруднения в силу недостатка опыта поездок в места, где такой способ распашки распространен.


    Пример 5

    Тело пингвинов покрыто очень мощным плотным слоем контурных перьев, под которыми расположен толстый слой пуховых перьев. При этом пингвины, в отличие от других птиц, меняют пуховые перья все разом, а не постепенно в течение всей жизни. Объясните, почему у пингвинов в ходе эволюции сформировались такие особенности пухового слоя перьев и как эти особенности повышают их приспособленность к условиям окружающей среды.

    Элементы ответа:
    1. Пингвины живут в холодных условиях, поэтому мощный пуховой слой (пуховые перья особого строения) им необходим в качестве термоизоляции.

    2. Если бы пуховые перья менялись постепенно, это приводило бы к нарушению плотности контурных перьев, что, в свою очередь, приводило к намоканию перьев при плавании.

    3. Поэтому у пингвинов выработалась смена всех пуховых перьев разом, чтобы период смены был как можно короче.

    Примечание: Абитуриент может не понять смысл вопроса. Нужно уделить время этому заданию, представить веер ответов. Лучше — пользуясь дополнительной информацией. Необходимо проговорить, что линька у пингвинов длится около 20 дней — это короткое время, в течение которого птицы голодают и скапливаются в стаи, чтобы согреться. Пуховые перья выталкиваются новыми перьями. Если словосочетание «все разом» будет понято абитуриентом как «очень быстро» (что вполне вероятно), возникнут трудности с ответом.


    Пример 6

    Что происходит с признаками и характеристиками организмов при дивергентном видообразовании? Какие движущие силы эволюции лежат в основе этого процесса? Какая форма естественного отбора лежит в основе этого процесса?

    Элементы ответа:
    1. При дивергенции происходит расхождение признаков.

    2. Дивергенция обусловлена наследственной изменчивостью, борьбой за существование и естественным отбором.

    3. Движущая форма естественного отбора, ведущая к полиморфизму.

    Примечание: Термин полиморфизм будет часто встречаться в заданиях 2018 года. В некоторых вопросах подобного типа нужно будет говорить о дизруптивной форме отбора.


    Пример 7

    Почему биологический регресс часто ведет к вымиранию вида? Ответ обоснуйте, приведите не менее четырех аргументов.

    Элементы ответа:
    1. При биологическом регрессе резко сокращается численность вида в связи с понижением адаптации организмов при изменении условий среды.

    2. Происходит уменьшение ареала за счет уменьшения численности.

    3. Возникает близкородственное скрещивание, которое приводит к проявлению вредных мутаций и гибели организмов.

    4. Случайные факторы повышают вероятность вымирания вида.

    Примечание: Задание требует умения оперировать понятиями «регресс», «адаптация», «ареал», «мутации». Оно несложное, однако абитуриенты не всегда могут привести именно четыре полных критерия. Возможный веер ответов: организмы не успевают приспособиться к изменяющимся условиям среды; они не выдерживают конкуренции с другими видами; в результате они отсеиваются естественным отбором; из-за снижения численности происходит сокращение ареала. Каждая из этих формулировок вполне соответствует алгоритму, приведенному в критериях.

    ЕГЭ. Биология. Новый полный справочник для подготовки к ЕГЭ

    Вниманию учащихся и учителей предлагается новое учебное пособие, которое поможет успешно подготовиться к единому государственному экзамену по биологии. Справочник содержит весь теоретический материал по курсу биологии, необходимый для сдачи ЕГЭ. Он включает в себя все элементы содержания, проверяемые контрольно-измерительными материалами, и помогает обобщить и систематизировать знания и умения за курс средней (полной) школы. Теоретический материал изложен в краткой, доступной форме. Каждый раздел сопровождается примерами тестовых заданий, позволяющими проверить свои знания и степень подготовленности к аттестационному экзамену. Практические задания соответствуют формату ЕГЭ. В конце пособия приводятся ответы к тестам, которые помогут школьникам и абитуриентам проверить себя и восполнить имеющиеся пробелы. Пособие адресовано школьникам, абитуриентам и учителям.

    Купить
    Пример 8

    Докажите, что большинство современных птиц находится в состоянии биологического прогресса (с учетом особенностей птиц).

    Элементы ответа:
    1. Большое видовое разнообразие, обусловленное разнообразием и адаптациями к своим экологическим нишам.

    2. Высокая внутривидовая численность особей, связанная со сложным поведением (заботой о потомстве, перелетами, строительством разнообразных гнезд и т.д.).

    3. Широкий ареал обитания, обусловленный теплокровностью и способностью к полету.

    Примечание: В данном примере не следует говорить о четырехкамерном сердце, двойном дыхании, клюве без зубов и прочих приспособлениях к полету, поскольку об этом не спрашивают. Теплокровность правильно упоминается как адаптациия к переживанию различных условий среды. Ответ на это задание, как и в предыдущем примере, поддается алгоритмизации: все, что сокращается при регрессе, увеличивается при прогрессе, и наоборот.


    Пример 9

    Опишите состав первичной атмосферы Земли и условия, при которых происходил абиогенный синтез первых органических веществ. Какие вещества синтезировали Миллер и Юри в своем эксперименте? Почему эти вещества не образуются в настоящее время, например при извержениях вулканов?

    Элементы ответа:
    1. Первичная атмосфера Земли содержала водяной пар, аммиак, водород и метан.

    2. Условиями для синтеза были электрические разряды и высокая температура.

    3. В результате опыта ученые получили несколько аминокислот, мочевину и молочную кислоту.

    4. Образующиеся органические вещества сразу поглощаются микроорганизмами или окисляются кислородом атмосферы.

    Примечание: Очевидно, что в ответе должно быть 4 критерия. Данный пример показывает, что необходимо знать имена некоторых ученых и их работы.


    Пример 10

    Клевер произрастает на лугу, опыляется шмелями. Какие биотические факторы могут привести к сокращению численности популяции клевера?

    Элементы ответа:
    1. Уменьшение численности шмелей.

    2. Увеличение численности растительноядных животных.

    3. Размножение растений-конкурентов (злаков и др.).

    Примечание: В задании могут быть приведены другие критерии:

    1. Вытаптывание клевера коровами.

    2. Разрушение гнезд шмелей птицами.

    3. Уничтожение яиц шмелей насекомыми-наездниками и т.д.


    Пример 11

    Какие растения в природных условиях получают минеральное питание не из почвы, и поясните — как?

    Элементы ответа:
    1. Эпифиты (например, орхидеи) — с осадками из разлагающихся растительных остатков, скапливающихся в трещинах коры деревьев, развилках ветвей и др.

    2. Многие бобовые (например, горох, фасоль) и другие растения (ольха), имеющие симбиоз с азотфиксирующими бактериями или актиномицетами, получают соединения азота главным образом не из почвы.

    3. Паразиты (например, петров крест, заразиха) и полупаразит (например, Омела) получают элементы минерального питания из растений-хозяев.

    4. Насекомоядные растения (например, росянка, венерина мухоловка) — из своих жертв.

    5. Водные растения (например, элодея, ряска) — поглощают элементы минерального питания из окружающей водной толщи.

    Примечание: Это сложное задание, и, чтобы на него ответить, нужно владеть большим объемом информации. Подобные вопросы по экологии должны быть хорошо проработаны, желательно в последний месяц перед экзаменом (они легко забываются). Очевидным преимуществом эпифитов, особенно в сильно заросшей деревьями местности, является возможность не зависеть от грунта, а находиться ближе к источнику света. Полупаразиты характеризуются слабым развитием корневой системы и почти пол¬ным отсутствием корневых волосков. С помощью корневых присосок они получают воду и питательные вещества из корней растения-хозяина.


    Пример 12

    Объясните, почему для возобновления вида орлана-белохвоста достаточно двух яиц в кладке, а в кладке соловья — 6-7 яиц.

    Элементы ответа:
    1. Высокой плодовитостью отличаются виды, у которых велика гибель особей в природе.

    2. У соловья избыточное производство яиц как бы покрывает их возможную гибель (гнездовой тип птенцов, много врагов, длительные перелеты на места зимовья и др.).

    3. У видов с хорошо развитой заботой о потомстве гибель птенцов невелика (более зрелые птенцы, отсутствие естественных врагов, оседлый образ жизни).

    Примечание: Задание требует от абитуриентов как минимум некоторого представления об орлане-белохвосте. Условие отчасти наводит на правильный ответ — по крайней мере, учащиеся могут сравнить размеры птиц, предположить заботу о потомстве у орланов, и, таким образом, вывести первый критерий.

    ЕГЭ. Биология. Большой сборник тематических заданий для подготовки к единому государственному экзамену

    Вниманию учащихся и учителей предлагается новое учебное пособие, которое поможет успешно подготовиться к единому государственному экзамену по биологии. Сборник содержит вопросы, подобранные по разделам и темам, проверяемым на ЕГЭ, и включает задания разных типов и уровней сложности. В конце пособия приводятся ответы на все задания. Предлагаемые тематические задания помогут учителю организовать подготовку к единому государственному экзамену, а учащимся — самостоятельно проверить свои знания и готовность к сдаче выпускного экзамена. Книга адресована учащимся, учителям и методистам.

    Купить
    Пример 13

    Объясните влияние плотности водной среды обитания на «живые организмы» биоценозов.

    Элементы ответа:
    1. Свет проникает в воду на небольшую глубину, поэтому растительные организмы могут существовать только в верхних слоях (до 150-200 м).

    2. Плотность водной среды оказывает влияние на обтекаемую форму тела и сильную мускулатуру быстро передвигающихся животных.

    3. Плотность среды облегчает вес организмов и создает возможность постоянно находиться в толще среды (планктон).

    4. Наличие планктона делает возможным фильтрационный тип питания многих животных.

    Примечание: Критерий № 4 едва ли имеет прямое отношение к вопросу, поскольку указано не прямое влияние воды на жизнедеятельность организмов. Возможный веер ответов: у водных растений слабо развитая механическая ткань и высокая плавучесть; у животных развиты такие приспособления, как слизь на коже, плавники, адаптация к разным глубинам и так далее. Задания по теме «Экология организмов» требуют умения применять знания в измененной ситуации. В вопросах встречается влияние света, влажности, солености и других факторов.


    Пример 14

    Укажите не менее четырех факторов водной среды обитания. Объясните их роль в жизни организмов.

    Элементы ответа:
    1. Плотность воды определяет ее выталкивающую силу (распределение организма по разным глубинам).

    2. Температурный режим более сглажен, нет чрезмерно высоких и низких температур.

    3. Ограниченное количество кислорода. Бывают заморы в водоемах по разным причинам.

    4. Солевой состав ограничивает распространение пресноводных и морских обитателей.

    Примечание: Подобные типы заданий могут касаться наземно-воздушной, почвенной, внутриорганизменной сред обитания. Речь идет о физико-химических свойствах и их роли в жизни организмов. Абитуриент ответит правильно, если такие свойства им хорошо изучены.


    Читайте также:

    Реальное егэ по биологии. Подготовка к ЕГЭ по биологии онлайн — Материалы

    В 2017 году ЕГЭ по биологии ждут большие изменения. Предстоящая реформа проводится в соответствии с планами министерства образования, обнародованных еще несколько лет назад. Цель реформирования – уйти от привычной тестовой системы, которая, по словам самих чиновников, дает достаточно ненадежные результаты. И в 2017 году биология стала очередным предметом, из которого убирают все вопросы-тесты.

    Дата проведения

    • 03.2017г. – первый этап, досрочный;
    • 04.2017г. – резервный день первого этапа;
    • 06.2017г. – второй этап, основной;
    • 06.2017г. – резервный день второго этапа;
    • 06.2017г. – резервный день по всем предметам.

    Изменения в ЕГЭ по биологии в 2017 году

    В 2017 году школьников ждут реформы сразу по трем предметам: химии, физике, биологии. Какие изменения внесены в ЕГЭ по биологии? В первую очередь это удаление тестовой части и замена ее на блоки с краткими ответами.

    • Изменение количества заданий. В ЕГЭ 2017 будет 28 заданий, а не 40, как это было раньше.
    • Введение заданий новых типов. Вместо заданий с выбором одного правильного ответа из четырех предложенных вариантов в экзамен вошли задачи на дополнение или восстановление таблиц и схем. Проверка навыков работы с графическим материалом – рисунками, схемами, таблицами, графиками – становится важной частью экзамена по биологии.
    • Увеличение продолжительности экзамена. Время, отведенное на ЕГЭ по биологии 2017, увеличится до 210 минус (против 180 в прошлом году).
    • Уменьшение проходного балла. Первичный/пороговый балл по биологии в 2017 году снижен до 59 (против 61 в прошлом году).

    Структура экзамена:

    • Рассмотрение данных в таблице или графике – 1 задание
    • Заполнение таблиц — 1 задание
    • Заполнение схем – 1 задание
    • Решение задач по генетике/цитологии – 2 задания
    • Определение последовательности в процессах/явлениях – 3 задание
    • Поиск соответствий – 6 заданий

    Примерная структура экзамена соблюдена во всех базах и демонстрационных версиях ЕГЭ, а что существенно облегчает подготовку. Особо стоит отметить, что новая модель КИМ сходна с вариантом ОГЭ по биологии, сдаваемом в девятом классе. Так что будущие выпускники могут не беспокоиться – структура экзамена не окажется чем-то принципиально новым для них.

    Оценка за ЕГЭ

    Минимальное количество баллов, которое необходимо набрать на биологии – 36, а первичный балл равняется 59. За правильные ответы на некоторые вопросы ученик может получить один балл, а за некоторые — три.

    • Вопросы 1, 3, 6 – ответы на них оцениваются в 1 балл;
    • Вопросы 2, 4, 5, 7, 8, 9, 10, 11, 12, 13, 14, 15, 16, 17, 18, 19, 20, 21, 22 – за правильные ответы можно получить 2 балла;
    • Вопросы 23, 24, 25, 26, 27, 28 – правильные ответы приносят 3 балла.

    По предупреждениям ФИПИ, некоторые вопросы осознанно усложнены. Ответы на них по силам лишь тем, кто знает биологию на «отлично». Такие вопросы потребуют не просто краткого ответа, а приведения аргументов в пользу выбранной версии и подробного пояснения хода мыслей.

    Как готовиться к экзамену

    Чтобы помочь школьникам легко и беспроблемно сдать ЕГЭ по биологии, ФИПИ публикует вспомогательные и методические материалы.

    Подготовка к ЕГЭ по биологии 2017 должна обязательно включать в себя работу с демоверсиями экзамена. На сегодняшний день существует 36 вар вариантов КИМов, причем все они со сходной структурой, то есть для подготовки можно взять любой из предложенных ФИПИ вариантов.

    Методисты уверяют – подготовка к экзамену займет довольно продолжительное время, и для успешного освоения материала лучше всего составить график и придерживаться его. В подготовке школьникам наверняка помогут созданные специалистами ФИПИ рекомендации:

    1. Ориентируясь на опубликованные ФИПИ темы, составьте список подразделов, которые необходимо повторить. Исходя из объема и сложности темы, составьте график их изучения, при этом на сложные темы необходимо отвести больше времени, чем на простые и легкие.
    2. Изучать материал лучше всего по школьному учебнику. В качестве дополнительных источников можно использовать рекомендованные ведомством методические материалы и информацию из всемирной сети.
    3. Чтобы облегчить изучение темы и облегчить запоминание материала, сведите данные по изучаемой теме в таблицу или схематичный конспект.
    4. Используйте опубликованные на сайте ФИПИ КИМы для подготовки. Благодаря этому вы сможете обнаружить пробелы в знаниях по определенным темам.
    5. Изучая новые темы, периодически возвращайтесь к старым, повторяя и освежая в памяти пройденный материал.

    Демоверсии экзамена помогут в подготовке к экзаменам. Помимо уже упомянутой возможности обнаружить слабо изученные темы, онлайн-версии помогают познакомиться с самой структурой экзамена.

    Вывод

    Рассматривая все внесенные изменения, приходим к малоутешительному (для части школьников) выводу – ЕГЭ по биологии в 2017 году будет сдать значительно сложнее, чем в 2016. Сложнее будет получить как минимальное количество баллов, то есть достичь порога проходимости, так и набрать максимальное значение. Пропадает шанс получить баллы за угадывание, придется действительно готовиться к экзамену и досконально изучать предмет.

    Биология – это довольно обширная область знаний, которая включает в себя строение и различные процессы, как в человеке, так и животных и растениях. В связи с этим, для успешного преодоление итогового экзамена необходимо владеть обширным кругом знаний. При том, что строение организмов различных видов и классов похожи, у них есть и различия, которые необходимо точно помнить, отвечая на тестовые вопросы. В связи с такой сложностью ЕГЭ по биологии необходимо готовиться ещё тщательнее.

    Способы подготовки к ЕГЭ по биологии

    К любому ЕГЭ есть множество способов подготовиться. Отличаются между собой лишь языковые технические и гуманитарные предметы. Но основные направления подготовки сохряняются. Так что ЕГЭ по биологиине должно выпадать из общего курса подготовки к единому экзамену. Необходимо комплексно подходить к данному вопросу.

    Не стоит забывать о школьных занятиях. В целом, школа может неплохо подготовить ребёнка к экзамену. Всё зависит от его способностей, памяти и психики. Поэтому не стоит списывать школу со счетов при подготовки к тяжелому ЕГЭ по биологии.

    При выборе курсов подготовки к ЕГЭстоит обратить внимание на отзывы и время существования конторы. Если курсы открылись недавно, то у них меньше опыта и есть вероятность обмана или закрытия курсов на полпути подготовки. Но к молодым курсам подготовки к ЕГЭ можно обратиться, если они заманивают низками ценами и у них отсутствует предоплата. Но всё же внимательно побеседуйте с преподавателями и решите: смогут ли они вас подготовить к ЕГЭ. На результатах ЕГЭ экономить не стоит, но нельзя же полностью отказываться от услуг молодых специалистов. Главное не потратить свои средства впустую.

    Репетиторы по биологии смогут подтянуть уровень знания, в том числе помогут готовиться к ЕГЭ. Но выбирать репетитора надо тщательнее, чем курсы подготовки. Курсы получили государственную лицензию и прошли проверки, в то время как частным репетиторством может заниматься любой желающий. Так что поищите отзывы и рекомендации, прежде чем начинать платить такому специалисту. Ведь ваше время ограничено, и тратить его впустую не стоит. Так что лучше потратить пару дней на поиски хорошего специалиста, чем заниматься с тем, кто не может научить важным вещам.

    Пробные онлайн тесты ЕГЭ по биологии

    На образовательном портале сайт размещены пробные варианты ЕГЭ по биологии, которые доступны всем посетителям. На нашем сайте, можно пройти тест неограниченное количество раз, без каких либо временных рамок. Это позволит выявить пробелы в знаниях и заняться их устранением. Так что не стоит забывать про самоподготовку. Ведь тренировочные ЕГЭ погружают вас в атмосферу тестового экзамена, делая его более привычным. Повторение тестов поможет избавиться от стресса на самом ЕГЭ, а это повысить итоговую оценку. В стрессом состоянии память работает хуже и возможны ошибки. По этой причине каждому будет полезно уделить время для самостоятельной подготовки с помощью онлайн тестов ЕГЭ по биологии на образовательном портале Uchistut.ru.

    Видеокурс «Получи пятерку» включает все темы, необходимые для успешной сдачи ЕГЭ по математике на 60-65 баллов. Полностью все задачи 1-13 Профильного ЕГЭ по математике. Подходит также для сдачи Базового ЕГЭ по математике. Если вы хотите сдать ЕГЭ на 90-100 баллов, вам надо решать часть 1 за 30 минут и без ошибок!

    Курс подготовки к ЕГЭ для 10-11 класса, а также для преподавателей. Все необходимое, чтобы решить часть 1 ЕГЭ по математике (первые 12 задач) и задачу 13 (тригонометрия). А это более 70 баллов на ЕГЭ, и без них не обойтись ни стобалльнику, ни гуманитарию.

    Вся необходимая теория. Быстрые способы решения, ловушки и секреты ЕГЭ. Разобраны все актуальные задания части 1 из Банка заданий ФИПИ. Курс полностью соответствует требованиям ЕГЭ-2018.

    Курс содержит 5 больших тем, по 2,5 часа каждая. Каждая тема дается с нуля, просто и понятно.

    Сотни заданий ЕГЭ. Текстовые задачи и теория вероятностей. Простые и легко запоминаемые алгоритмы решения задач. Геометрия. Теория, справочный материал, разбор всех типов заданий ЕГЭ. Стереометрия. Хитрые приемы решения, полезные шпаргалки, развитие пространственного воображения. Тригонометрия с нуля — до задачи 13. Понимание вместо зубрежки. Наглядное объяснение сложных понятий. Алгебра. Корни, степени и логарифмы, функция и производная. База для решения сложных задач 2 части ЕГЭ.

    Экзамен по биологии относится к числу выборочных и сдавать ее будут те, кто уверен в знаниях. ЕГЭ по биологии считается сложным предметом, так как проверяются знания, накопленные за все годы изучения.

    Задания ЕГЭ по биологии подобраны разнотипные, для их решения необходимы уверенные знания по основным темам школьного курса биологии. На основе педагоги разработали свыше 10 тестовых заданий по каждой теме.

    Темы, которые нужно изучить при выполнении заданий смотрите от ФИПИ. Для каждого задания прописан свой алгоритм действий, который поможет при решении задач.

    Изменения в КИМ ЕГЭ 2019 г. по биологии:

    • Изменена модель задания в линии 2. Вместо задания с множественным выбором на 2 балла включено задание на работу с таблицей на 1 балл.
    • Максимальный первичный балл уменьшился на 1 и составил 58 баллов.

    Структура заданий ЕГЭ по биологии:

    • Часть 1 – это задания с 1 по 21 с кратким ответом, на выполнение отводится примерно до 5 минут.

    Совет : внимательно читайте формулировки вопросов.

    • Часть 2 – это задания с 22 по 28 с развернутым ответом, на выполнение отводится примерно 10-20 минут.

    Совет : литературно излагай свои мысли, отвечай на вопрос подробно и всесторонне, давай определение биологическим терминам, даже если этого не требуют в заданиях. В ответе должен быть план, не писать сплошным текстом, а выделять пункты.

    Что требуются от ученика на экзамене?

    • Умение работать с графической информацией (схемы, графики, таблицы) – ее анализ и использование;
    • Множественный выбор;
    • Установление соответствия;
    • Установление последовательности.


    Баллы за каждое задание по биологии ЕГЭ

    Для того, чтобы получить наивысшую оценку по биологии, необходимо набрать 58 первичных баллов, которые будут переведены в сто по шкале.

    • 1 балл — за 1, 2, 3, 6 задания.
    • 2 балла — 4, 5, 7-22.
    • З балла — 23-28.


    Как подготовиться к тестовым заданиям по биологии
    1. Повторение теории.
    2. Правильное распределение времени на каждое задание.
    3. Решение практических задач по несколько раз.
    4. Проверка уровня знаний путем решения тестов онлайн.

    Регистрируйся, занимайся и получай высокий балл!

    Биология ЕГЭ 2018 Тематический тренинг Все типы заданий Кириленко

    Тематический тренинг ЕГЭ 2018 Кириленко по биологии содержит все типы заданий: 800 заданий, разделенных по темам – уровню сложности; задания с использованием рисунков, задания с множественным выбором ответов, задания на установление последовательности биологическ. процессов или явлений. Содержит также ответы к заданиям; пояснения – комментарии к заданиям повышенной трудности. Предназначено учебное пособие для начального этапа под-ки к ЕГЭ. Тренинг способствует систематизации изученного материала, закреплению навыков выполнения заданий, аналогичных экзаменационным. Пособие даст полное представление о предстоящем экзамене. Тренинг адресован учащимся 11 классов, учителям, методистам, преподавателям курсов по под-ке к ЕГЭ.

    – Содержание –

    Методические рекомендации обучающимся 7
    1. Биологические термины – понятия (дополнение схемы) 17
     Биология как наука. Мет-ды научного познания. Клетка – биологическая система 17
    Система – многообразие органического мира. Вир-усы. Бактерии. Грибы. Лишайники 27
    Система – многообразие органического мира. Раст-ния 31
    Система – многообразие органического мира. Жив-тные 36
    Человек – его здоровье 42
    Эволюция живой природы 48
    1.8. Экология 53
    2. Множественный выбор отв-тов (с рисунком – без рисунка) 58
    Биология – наука. Методы научного позн-ния 58
    Клетка – биологическая система 61
    Организм – биологическая система 67
    Система – многообразие органического мира 73
    Организм человека – его здоровье 84
    Экосистемы – присущие им закономерности. Ср-ды жизни. Биосфера 90
    Воспроизведение организмов. Онтогенез. Законом-рности наследственности -изменчивости. Селекция – биотехнология 94
    Эволюция живой природы. Микроэв-люция. Макроэволюция. Происхождение человека 97
    Решение биологической задачи 102
    Задания с множественным выб-ром ответов 106
     Клетка – биологическая система 106
    Организм – биологическая система 112
    Растения. Бактерии. Грибы. Лиш-йники 117
     Животные 122
    Организм человека – его здоровье 127
    Эволюция живой природы 134
    Экосистемы – присущие им закономерности. Ср-ды жизни. Биосфера 139
    Установление последовательности 144
    Многообразие организмов. Основные системат-ческие категории, их соподчинённость 144
    Клетка -биологическая система 147
     Организм – биологическая система 151
    Растения. Бактерии. Грибы. Лиш-йники 154
    Животные 157
    Человек – его здоровье 160
    Эволюция живой природы 165
    Экосистемы – присущие им закономерности 168
    6. Дополнение таблицы ( рисунком – без рисунка) 172
    Анализ информации, представленной – табличной или графической ф-рме 219
    Задания со свободным разв-рнутым ответом 228
     Практико-ориентированные задания 228
    Ответы 276
    Приложения 334
    Словарь терминов 334
    Вклад некоторых учёных – развитие биологии 354
    Основные теории, законы, прав-ла
    и принципы биологии 364
    Список литературы 370

    Размер файла: 7 Мб; Формат: pdf

    Извините! Ссылки для скачивания удалены по требованию изд-ва!

    Вместе с «тематическим тренингом со всеми типами заданий Кириленко к ЕГЭ 2018 по биологии» скачивают:

    Admin

    Решу егэ по биологии. Каталог заданий ЕГЭ по биологии Гущин с пояснениями

    Вариант № 2004478

    Де­мон­стра­ци­он­ная вер­сия ЕГЭ-2018 по биологии.

    При выполнении заданий с кратким ответом впишите в поле для ответа цифру, которая соответствует номеру правильного ответа, или число, слово, последовательность букв (слов) или цифр. Ответ следует записывать без пробелов и каких-либо дополнительных символов. Единицы измерений писать не нужно.

    Если вариант задан учителем, вы можете вписать или загрузить в систему ответы к заданиям с развернутым ответом. Учитель увидит результаты выполнения заданий с кратким ответом и сможет оценить загруженные ответы к заданиям с развернутым ответом. Выставленные учителем баллы отобразятся в вашей статистике.


    Версия для печати и копирования в MS Word

    Рассмотрите предложенную схему классификации видов изменчивости. Запишите в ответе пропущенный термин, обозначенный на схеме знаком вопроса.

    Ответ:

    Выберите два верных ответа из пяти и запишите в ответ цифры, под которыми они указаны. Цитогенетический метод используют для определения

    1) степени влияния среды на формирование фенотипа

    2) наследования сцепленных с полом признаков

    3) кариотипа организма

    4) хромосомных аномалий

    5) возможности проявления признаков у потомков

    Ответ:

    В соматической клетке тела рыбы 56 хромосом. Какой набор хромосом имеет сперматозоид рыбы? В ответе запишите только количество хромосом.

    В ДНК на долю нуклеотидов с аденином приходится 18%. Определите процентное содержание нуклеотидов с цитозином, входящих в состав молекулы. В ответе запишите только соответствующее число.

    Ответ:

    Все перечисленные ниже признаки, кроме двух, используются для описания изображённой на рисунке клетки. Определите два признака, «выпадающих» из общего списка, и запишите в таблицу цифры, под которыми они указаны.

    1) наличие хлоропластов

    2) наличие гликокаликса

    3) способность к фотосинтезу

    4) способность к фагоцитозу

    5) способность к биосинтезу белка

    Все перечисленные ниже признаки, кроме двух, можно использовать для описания молекулы ДНК. Определите два признака, «выпадающих» из общего списка, и запишите в таблицу цифры, под которыми они указаны.

    1) состоит из двух полинуклеотидных цепей, закрученных в спираль

    2) переносит информацию к месту синтеза белка

    3) в комплексе с белками строит тело рибосомы

    4) способна самоудваиваться

    5) в комплексе с белками образует хромосомы

    Ответ:

    Установите соответствие между процессами и этапами энергетического обмена: к каждой позиции, данной в первом столбце, подберите соответствующую позицию из второго столбца.

    Ответ:

    Определите соотношение фенотипов у потомков при моногибридном скрещивании двух гетерозиготных организмов при полном доминировании. Ответ запишите в виде последовательности цифр, показывающих соотношение получившихся фенотипов, в порядке их убывания.

    Ответ:

    Все приведённые ниже термины, кроме двух, используются для описания полового размножения организмов. Определите два термина, «выпадающих» из общего списка, и запишите в таблицу цифры, под которыми они указаны.

    3) оплодотворение

    4) овогенез

    5) почкование

    Ответ:

    Установите соответствие между примером биологического явления и формой изменчивости, которую он иллюстрирует: к каждой позиции, данной в первом столбце, подберите соответствующую позицию из второго столбца.

    Запишите в ответ цифры, рас­по­ло­жив их в порядке, со­от­вет­ству­ю­щем буквам:

    Установите соответствие между структурами и зародышевыми листками, обозначенными на рисунке цифрами 1, 2, из которых эти структуры формируются: к каждой позиции, данной в первом столбце, подберите соответствующую позицию из второго столбца.

    Запишите в ответ цифры, рас­по­ло­жив их в порядке, со­от­вет­ству­ю­щем буквам:

    Ответ:

    Если в процессе эволюции у животного сформировался головной мозг, изображённый на рисунке, то для этого животного характерны

    1) четырёхкамерное сердце

    2) наружное оплодотворение

    3) кожные покровы с чешуйками или щитками

    4) постоянная температура тела

    5) ячеистые лёгкие

    6) развитие зародыша в матке

    Известно, что бактерия туберкулёзная палочка – аэробный, микроскопический, болезнетворный организм. Выберите из приведённого ниже текста три утверждения, относящиеся к описанию перечисленных выше признаков бактерии.

    Цифры ука­жи­те в по­ряд­ке возрастания.

    Ответ:

    Установите соответствие между функциями и органами растения, которые выполняют эти функции: к каждой позиции, данной в первом столбце, подберите соответствующую позицию из второго столбца.

    Запишите в ответ цифры, рас­по­ло­жив их в порядке, со­от­вет­ству­ю­щем буквам:

    Ответ:

    Установите последовательность расположения систематических таксонов растения, начиная с самого крупного таксона. Запишите в таблицу соответствующую последовательность цифр.

    1) Мятлик луговой

    3) Покрытосеменные

    4) Однодольные

    5) Растения

    6) Злаковые

    Ответ:

    Выберите три верных ответа из шести и запишите в таблицу цифры, под которыми они указаны.

    Во время бега в организме человека

    Цифры ука­жи­те в по­ряд­ке возрастания.

    1) растёт синтез желчи клетками печени

    2) ускоряется процесс биосинтеза белка в скелетных мышцах

    3) снижается количество лейкоцитов в плазме

    4) усиливается приток крови к коже

    5) возрастает потоотделение

    6) повышается возбудимость нервной системы

    Выберите три верно обозначенные подписи к рисунку «Строение уха». Запишите в таблицу цифры, под которыми они указаны.

    Цифры ука­жи­те в по­ряд­ке возрастания.

    1) наружный слуховой проход

    2) барабанная перепонка

    3) слуховой нерв

    5) полукружный канал

    Ответ:

    Установите соответствие между характеристиками и типами ткани человека: к каждой позиции, данной в первом столбце, подберите соответствующую позицию из второго столбца.

    Запишите в ответ цифры, рас­по­ло­жив их в порядке, со­от­вет­ству­ю­щем буквам:

    Ответ:

    Установите последовательность процессов, происходящих в пищеварительной системе человека при переваривании пищи. Запишите в таблицу соответствующую последовательность цифр.

    1) интенсивное всасывание воды

    2) набухание и частичное расщепление белков

    3) начало расщепления крахмала

    4) всасывание аминокислот и глюкозы в кровь

    5) расщепление всех биополимеров пищи до мономеров

    Ответ:

    Прочитайте текст. Выберите три предложения, в которых даны описания морфологического критерия вида сосны обыкновенной. Запишите в таблицу цифры, под которыми они указаны.

    (1) Сосна обыкновенная — светолюбивое растение. (2) Она имеет высокий стройный ствол, крона формируется только вблизи верхушки. (3) Сосна растёт на песчаных почвах, меловых горах. (4) У неё хорошо развиты главный и боковые корни, листья игловидные, по две хвоинки в узле на побеге. (5) На молодых побегах развиваются зеленовато-жёлтые мужские шишки и красноватые женские шишки. (6) Пыльца переносится ветром и попадает на женские шишки, где происходит оплодотворение.

    Ответ:

    Установите соответствие между примерами объектов и методами изучения эволюции, в которых используются эти примеры: к каждой позиции, данной в первом столбце, подберите соответствующую позицию из второго столбца.

    Запишите в ответ цифры, рас­по­ло­жив их в порядке, со­от­вет­ству­ю­щем буквам:

    Ответ:

    Выберите три верных ответа из шести и запишите в таблицу цифры, под которыми они указаны. Устойчивость экосистемы влажного экваториального леса определяется. Цифры ука­жи­те в по­ряд­ке возрастания.

    1) большим видовым разнообразием

    2) отсутствием редуцентов

    3) большой численностью хищников

    4) разветвлёнными пищевыми сетями

    5) колебанием численности популяций

    6) замкнутым круговоротом веществ

    Ответ:

    Установите соответствие между примерами и экологическими факторами, которые этими примерами иллюстрируются: к каждой позиции, данной в первом столбце, подберите соответствующую позицию из второго столбца.

    Запишите в ответ цифры, рас­по­ло­жив их в порядке, со­от­вет­ству­ю­щем буквам:

    Ответ:

    Установите последовательность эволюционных процессов, происходивших на Земле, в хронологическом порядке. Запишите в таблицу соответствующую последовательность цифр.

    1) выход организмов на сушу

    2) возникновение фотосинтеза

    3) формирование озонового экрана

    4) абиогенный синтез органических веществ

    5) появление клеточных форм жизни

    Ответ:

    Рассмотрите рисунок с изображением бабочки берёзовой пяденицы и определите тип приспособления, форму естественного отбора и направление эволюции, которые привели к формированию двух форм бабочек. Заполните пустые ячейки таблицы, используя термины, приведённые в списке. Для каждой ячейки, обозначенной буквами, выберите соответствующий термин из предложенного списка.

    Список терминов:

    1) идиоадаптация

    2) мимикрия

    3) конвергенция

    4) движущая

    5) ароморфоз

    6) маскировка

    7) стабилизирующая

    Проанализируйте таблицу «Структуры клетки». Заполните пустые ячейки таблицы, используя термины, приведённые в списке. Для каждой ячейки, обозначенной буквой, выберите соответствующий термин из предложенного списка.

    Структуры клетки

    Список терминов:

    1) гликолиз

    2) хлоропласт

    3) биосинтез белка

    4) митохондрия

    5) транскрипция

    7) цитоплазма

    8) клеточный центр

    Ответ:

    Проанализируйте таблицу «Выживание птенцов скворца в зависимости от количества яиц в кладке».

    Выживание птенцов скворца в зависимости от количества яиц в кладке

    Количество яиц вДоля выживших птенцов (в %)
    1100
    295
    390
    483
    580
    653
    740
    835
    932

    1) Оптимальное количество яиц в кладке, позволяющее сохранить численность скворцов, – 5.

    2) Гибель птенцов объясняется случайными факторами.

    3) Чем меньше в кладке яиц, тем эффективнее забота о потомстве.

    4) Чем больше птенцов в гнезде, тем чаще родители кормят каждого из птенцов.

    5) Количество яиц в кладке зависит от климатических факторов и наличия корма.

    Проанализируйте график скорости размножения молочнокислых бактерий.

    Выберите утверждения, которые можно сформулировать на основании анализа полученных результатов.

    Скорость размножения бактерий

    1) всегда прямо пропорциональна изменению температуры среды.

    2) зависит от ресурсов среды, в которой находятся бактерии.

    3) зависит от генетической программы организма.

    4) в интервале от 20 до 36 °С повышается.

    5) уменьшается при температуре выше 36 °С в связи с денатурацией части белков в клетке бактерии.

    Проанализируйте гистограмму состава рациона животного Z.

    По оси х расположены таксоны позвоночных животных, которыми питается животное Z, а по оси у – их количество в рационе. Выберите утверждения, которые можно сформулировать на основании анализа гистограммы.

    Животное Z относят к

    1) всеядным животным

    2) консументам II порядка

    3) производителям органических веществ

    4) полуводным животным

    5) обитателям тундры

    Ответ:

    Известно, что в плазме крови концентрация раствора солей в норме составляет 0,9%. В стеклянный стакан, заполненный раствором поваренной соли, поместили эритроциты. Сравните изображение нормального эритроцита в плазме (рис. А) и эритроцита в растворе (рис. Б). Объясните наблюдаемое явление. Определите концентрацию соли в стакане с раствором (более 0,9%, менее 0,9%, равна 0,9%).

    Известно, что в растительных клетках присутствуют два вида хлорофилла: хлорофилл a и хлорофилл b . Учёному, для изучения их структуры, необходимо разделить эти два пигмента. Какой метод он должен использовать для их разделения? На чём основан этот метод?

    Анализа результатов нарушения сцепленного наследования генов позволяет определить последовательность расположения генов в хромосоме и составить генетические карты. Результаты многочисленных скрещиваний мух дрозофил показали, что частота нарушения сцепления между генами А и В составляет 5%, между генами А и С — 11%, между генами С и В — 6%. Перерисуйте предложенную схему фрагмента хромосомы на лист ответа, отметьте на ней взаимное расположение генов А , В , С и укажите расстояние между ними. Какая величина принята за единицу расстояния между генами? Как она называется?

    Государственная итоговая аттестация 2019 года по биологии для выпускников 9 класса общеобразовательных учреждений проводится с целью оценки уровня общеобразовательной подготовки выпускников по данной дисциплине. В заданиях проверяются знания следующих разделов биологии:

    1. Роль биологии в формировании современной естественнонаучной картины мира, в практической деятельности людей.
    2. Клеточное строение организмов как доказательство их родства, единства живой природы.
    3. Признаки организмов. Одноклеточные и многоклеточные организмы. Царство Бактерии. Царство Грибы.
    4. Царство Растения.
    5. Царство Животные.
    6. Общий план строения и процессы жизнедеятельности. Сходство человека с животными и отличие от них. Размножение и развитие организма человека.
    7. Нейрогуморальная регуляция процессов жизнедеятельности организма.
    8. Опора и движение.
    9. Внутренняя среда.
    10. Транспорт веществ.
    11. Питание. Дыхание.
    12. Обмен веществ. Выделение. Покровы тела.
    13. Органы чувств.
    14. Психология и поведение человека.
    15. Соблюдение санитарно-гигиенических норм и правил здорового образа жизни. Приемы оказания первой доврачебной помощи.
    16. Влияние экологических факторов на организмы.
    17. Экосистемная организация живой природы. Биосфера. Учение об эволюции органического мира.
    В данном разделе вы найдёте онлайн тесты, которые помогут вам подготовиться к сдаче ОГЭ (ГИА) по биологии. Желаем успехов!

    Стандартный тест ОГЭ (ГИА-9) формата 2019-го года по биологии содержит две части. Первая состоит из 28 заданий с кратким ответом, вторая — из 4 заданий с развёрнутым ответом. В связи с этим в данном тесте представлена только первая часть (т.е. первые 28 заданий). Согласно текущей структуре экзамена, среди этих заданий варианты ответов предлагаются только в 22 заданиях. Но для удобства прохождения тестов администрация сайта сайт приняла решение предложить варианты ответов во всех заданиях. Однако для заданий, в которых варианты ответов составителями реальных контрольно измерительных материалов (КИМов) не предусмотрены, количество вариантов ответов было значительно увеличено, чтобы максимально приблизить наш тест к тому, с чем Вам придется столкнуться в конце учебного года.

    Стандартный тест ОГЭ (ГИА-9) формата 2019-го года по биологии содержит две части. Первая состоит из 28 заданий с кратким ответом, вторая — из 4 заданий с развёрнутым ответом. В связи с этим в данном тесте представлена только первая часть (т.е. первые 28 заданий). Согласно текущей структуре экзамена, среди этих заданий варианты ответов предлагаются только в 22 заданиях. Но для удобства прохождения тестов администрация сайта сайт приняла решение предложить варианты ответов во всех заданиях. Однако для заданий, в которых варианты ответов составителями реальных контрольно измерительных материалов (КИМов) не предусмотрены, количество вариантов ответов было значительно увеличено, чтобы максимально приблизить наш тест к тому, с чем Вам придется столкнуться в конце учебного года.

    Стандартный тест ОГЭ (ГИА-9) формата 2018-го года по биологии содержит две части. Первая состоит из 28 заданий с кратким ответом, вторая — из 4 заданий с развёрнутым ответом. В связи с этим в данном тесте представлена только первая часть (т.е. первые 28 заданий). Согласно текущей структуре экзамена, среди этих заданий варианты ответов предлагаются только в 22 заданиях. Но для удобства прохождения тестов администрация сайта сайт приняла решение предложить варианты ответов во всех заданиях. Однако для заданий, в которых варианты ответов составителями реальных контрольно измерительных материалов (КИМов) не предусмотрены, количество вариантов ответов было значительно увеличено, чтобы максимально приблизить наш тест к тому, с чем Вам придется столкнуться в конце учебного года.

    Стандартный тест ОГЭ (ГИА-9) формата 2018-го года по биологии содержит две части. Первая состоит из 28 заданий с кратким ответом, вторая — из 4 заданий с развёрнутым ответом. В связи с этим в данном тесте представлена только первая часть (т.е. первые 28 заданий). Согласно текущей структуре экзамена, среди этих заданий варианты ответов предлагаются только в 22 заданиях. Но для удобства прохождения тестов администрация сайта сайт приняла решение предложить варианты ответов во всех заданиях. Однако для заданий, в которых варианты ответов составителями реальных контрольно измерительных материалов (КИМов) не предусмотрены, количество вариантов ответов было значительно увеличено, чтобы максимально приблизить наш тест к тому, с чем Вам придется столкнуться в конце учебного года.

    Стандартный тест ОГЭ (ГИА-9) формата 2018-го года по биологии содержит две части. Первая состоит из 28 заданий с кратким ответом, вторая — из 4 заданий с развёрнутым ответом. В связи с этим в данном тесте представлена только первая часть (т.е. первые 28 заданий). Согласно текущей структуре экзамена, среди этих заданий варианты ответов предлагаются только в 22 заданиях. Но для удобства прохождения тестов администрация сайта сайт приняла решение предложить варианты ответов во всех заданиях. Однако для заданий, в которых варианты ответов составителями реальных контрольно измерительных материалов (КИМов) не предусмотрены, количество вариантов ответов было значительно увеличено, чтобы максимально приблизить наш тест к тому, с чем Вам придется столкнуться в конце учебного года.

    Стандартный тест ОГЭ (ГИА-9) формата 2018-го года по биологии содержит две части. Первая состоит из 28 заданий с кратким ответом, вторая — из 4 заданий с развёрнутым ответом. В связи с этим в данном тесте представлена только первая часть (т.е. первые 28 заданий). Согласно текущей структуре экзамена, среди этих заданий варианты ответов предлагаются только в 22 заданиях. Но для удобства прохождения тестов администрация сайта сайт приняла решение предложить варианты ответов во всех заданиях. Однако для заданий, в которых варианты ответов составителями реальных контрольно измерительных материалов (КИМов) не предусмотрены, количество вариантов ответов было значительно увеличено, чтобы максимально приблизить наш тест к тому, с чем Вам придется столкнуться в конце учебного года.

    Стандартный тест ОГЭ (ГИА-9) формата 2017-го года по биологии содержит две части. Первая состоит из 28 заданий с кратким ответом, вторая — из 4 заданий с развёрнутым ответом. В связи с этим в данном тесте представлена только первая часть (т.е. первые 28 заданий). Согласно текущей структуре экзамена, среди этих заданий варианты ответов предлагаются только в 22 заданиях. Но для удобства прохождения тестов администрация сайта сайт приняла решение предложить варианты ответов во всех заданиях. Однако для заданий, в которых варианты ответов составителями реальных контрольно измерительных материалов (КИМов) не предусмотрены, количество вариантов ответов было значительно увеличено, чтобы максимально приблизить наш тест к тому, с чем Вам придется столкнуться в конце учебного года.

    Стандартный тест ОГЭ (ГИА-9) формата 2016-го года по биологии содержит две части. Первая состоит из 28 заданий с кратким ответом, вторая — из 4 заданий с развёрнутым ответом. В связи с этим в данном тесте представлена только первая часть (т.е. первые 28 заданий). Согласно текущей структуре экзамена, среди этих заданий варианты ответов предлагаются только в 22 заданиях. Но для удобства прохождения тестов администрация сайта сайт приняла решение предложить варианты ответов во всех заданиях. Однако для заданий, в которых варианты ответов составителями реальных контрольно измерительных материалов (КИМов) не предусмотрены, количество вариантов ответов было значительно увеличено, чтобы максимально приблизить наш тест к тому, с чем Вам придется столкнуться в конце учебного года.

    Стандартный тест ОГЭ (ГИА-9) формата 2016-го года по биологии содержит две части. Первая состоит из 28 заданий с кратким ответом, вторая — из 4 заданий с развёрнутым ответом. В связи с этим в данном тесте представлена только первая часть (т.е. первые 28 заданий). Согласно текущей структуре экзамена, среди этих заданий варианты ответов предлагаются только в 22 заданиях. Но для удобства прохождения тестов администрация сайта сайт приняла решение предложить варианты ответов во всех заданиях. Однако для заданий, в которых варианты ответов составителями реальных контрольно измерительных материалов (КИМов) не предусмотрены, количество вариантов ответов было значительно увеличено, чтобы максимально приблизить наш тест к тому, с чем Вам придется столкнуться в конце учебного года.

    Стандартный тест ОГЭ (ГИА-9) формата 2016-го года по биологии содержит две части. Первая состоит из 28 заданий с кратким ответом, вторая — из 4 заданий с развёрнутым ответом. В связи с этим в данном тесте представлена только первая часть (т.е. первые 28 заданий). Согласно текущей структуре экзамена, среди этих заданий варианты ответов предлагаются только в 22 заданиях. Но для удобства прохождения тестов администрация сайта сайт приняла решение предложить варианты ответов во всех заданиях. Однако для заданий, в которых варианты ответов составителями реальных контрольно измерительных материалов (КИМов) не предусмотрены, количество вариантов ответов было значительно увеличено, чтобы максимально приблизить наш тест к тому, с чем Вам придется столкнуться в конце учебного года.

    Стандартный тест ОГЭ (ГИА-9) формата 2016-го года по биологии содержит две части. Первая состоит из 28 заданий с кратким ответом, вторая — из 4 заданий с развёрнутым ответом. В связи с этим в данном тесте представлена только первая часть (т.е. первые 28 заданий). Согласно текущей структуре экзамена, среди этих заданий варианты ответов предлагаются только в 22 заданиях. Но для удобства прохождения тестов администрация сайта сайт приняла решение предложить варианты ответов во всех заданиях. Однако для заданий, в которых варианты ответов составителями реальных контрольно измерительных материалов (КИМов) не предусмотрены, количество вариантов ответов было значительно увеличено, чтобы максимально приблизить наш тест к тому, с чем Вам придется столкнуться в конце учебного года.

    Стандартный тест ОГЭ (ГИА-9) формата 2016-го года по биологии содержит две части. Первая состоит из 28 заданий с кратким ответом, вторая — из 4 заданий с развёрнутым ответом. В связи с этим в данном тесте представлена только первая часть (т.е. первые 28 заданий). Согласно текущей структуре экзамена, среди этих заданий варианты ответов предлагаются только в 22 заданиях. Но для удобства прохождения тестов администрация сайта сайт приняла решение предложить варианты ответов во всех заданиях. Однако для заданий, в которых варианты ответов составителями реальных контрольно измерительных материалов (КИМов) не предусмотрены, количество вариантов ответов было значительно увеличено, чтобы максимально приблизить наш тест к тому, с чем Вам придется столкнуться в конце учебного года.

    Стандартный тест ОГЭ (ГИА-9) формата 2015-го года по биологии содержит две части. Первая состоит из 28 заданий с кратким ответом, вторая — из 4 заданий с развёрнутым ответом. В связи с этим в данном тесте представлена только первая часть (т.е. первые 28 заданий). Согласно текущей структуре экзамена, среди этих заданий варианты ответов предлагаются только в 22 заданиях. Но для удобства прохождения тестов администрация сайта сайт приняла решение предложить варианты ответов во всех заданиях. Однако для заданий, в которых варианты ответов составителями реальных контрольно измерительных материалов (КИМов) не предусмотрены, количество вариантов ответов было значительно увеличено, чтобы максимально приблизить наш тест к тому, с чем Вам придется столкнуться в конце учебного года.

    Стандартный тест ОГЭ (ГИА-9) формата 2015-го года по биологии содержит две части. Первая состоит из 28 заданий с кратким ответом, вторая — из 4 заданий с развёрнутым ответом. В связи с этим в данном тесте представлена только первая часть (т.е. первые 28 заданий). Согласно текущей структуре экзамена, среди этих заданий варианты ответов предлагаются только в 22 заданиях. Но для удобства прохождения тестов администрация сайта сайт приняла решение предложить варианты ответов во всех заданиях. Однако для заданий, в которых варианты ответов составителями реальных контрольно измерительных материалов (КИМов) не предусмотрены, количество вариантов ответов было значительно увеличено, чтобы максимально приблизить наш тест к тому, с чем Вам придется столкнуться в конце учебного года.

    Стандартный тест ОГЭ (ГИА-9) формата 2015-го года по биологии содержит две части. Первая состоит из 28 заданий с кратким ответом, вторая — из 4 заданий с развёрнутым ответом. В связи с этим в данном тесте представлена только первая часть (т.е. первые 28 заданий). Согласно текущей структуре экзамена, среди этих заданий варианты ответов предлагаются только в 22 заданиях. Но для удобства прохождения тестов администрация сайта сайт приняла решение предложить варианты ответов во всех заданиях. Однако для заданий, в которых варианты ответов составителями реальных контрольно измерительных материалов (КИМов) не предусмотрены, количество вариантов ответов было значительно увеличено, чтобы максимально приблизить наш тест к тому, с чем Вам придется столкнуться в конце учебного года.

    один правильный вариант .

    При выполнении заданий А1-А24 выберите только один правильный вариант .

    При выполнении заданий А1-А24 выберите только один правильный вариант .

    При выполнении заданий А1-А24 выберите только один правильный вариант .

    Среднее общее образование

    Биология

    ЕГЭ по биологии уже не за горами. В этом году проверка знаний будет несколько отличаться от экзаменов прошлых лет. Корпорация «Российский учебник» представляет серию вебинаров, в которых профессор МИОО Георгий Лернер производит разбор заданий и рассказывает, как должна проходить подготовка. На этот раз изучаем линию 26 из демонстрационного варианта, задания которой связаны с эволюционным учением и основами экологии – она является одной из наиболее сложных в ЕГЭ.

    Биология. ЕГЭ 2018 года. Особенности

    • С 2018 года будет уделяться особое внимание понятийному аппарату. Не исключено, что уже в будущем году в кодификаторах к экзамену пропишут терминологию, которой должен владеть учащийся.

    • Увеличится число вопросов, связанных с эрами и периодами, эволюцией растений и животных. Раньше они почти не встречались. Выпускникам необходимо изучить геохронологическую таблицу, быть готовым к заданиям на определение организма, относящегося к тому или иному периоду, к той или иной эре, на определение отпечатка, на расчет времени существования данного организма.

    • Увеличится число вопросов, посвященных происхождению жизни на Земле. Раньше их практически не было.

    • Почти исключится формулировка «Что произойдет, если исчезнут…». Некоторые последствия невозможно предугадать. Взамен абитуриентам будет предложено назвать факторы, которые могут привести к сокращению численности той или иной популяции.

    • В этом году абитуриентам не удастся ограничиться лаконичными ответами. Экзаменуемые должны показать знание предмета — для этого при подготовке необходимо уйти от «натаскивания».

    Варианты задания № 26, решения и пояснения
    Пример 1
    Какие ароморфозы обеспечили развитие древнейших организмов в архее и протерозое? Укажите не менее четырех ароморфных признаков и их значение в эволюции. Элементы ответа:
    Элементы ответа:
    1. Появление фотосинтеза обеспечило первичный синтез органических веществ из неорганических, накопление кислорода в воде и атмосфере, образование озонового экрана.

    2. Появление аэробного обмена веществ обеспечило синтез большого количества АТФ и снабжение организма энергией.

    3. Половой процесс привел к появлению у организмов разнообразных признаков — материала для эволюции.

    4. Появление многоклеточности привело к дифференциации клеток, тканей и органов.

    5. Появление эукариот обеспечило разнообразие царств живой природы.

    Примечание: Формулировки ответа могут быть разные, главное — правильное употребление терминов. Кроме того, если экзаменуемый не выполнит задание «Укажите», «Объясните», то потеряет 2 балла.

    Пример 2

    Популяция стабильна, если она имеет большую численность. Почему вероятность исчезновения малочисленных популяций выше, чем многочисленных?

    Элементы ответа:
    1. Вероятность гибели малочисленных популяций от воздействия неблагоприятных факторов внешней среды выше, чем у многочисленной популяции.

    2. Ослабевают связи между членами популяции (звуковая сигнализация, выделение химических веществ).

    3. Затруднена репродукция потомства.

    4. Близкородственное скрещивание особей в малочисленных популяциях приводит к появлению вредных рецессивных генов. При высокой численности популяция относительно гетерогенна.

    Примечание: В задании не указывается количество критериев, на которые нужно дать ответ, — следовательно, необходимо назвать не меньше трех. Ответ должен соответствовать смыслу вопроса, и если этого соответствия нет, то снижается балл. Для выполнения задания нужно указать события, которые ведут к исчезновению малых популяций. Критерий № 2 наиболее сложный для абитуриентов, он редко упоминается в алгоритме для решения подобной задачи.

    Пример 3

    У трески, щуки и многих других рыб количество выметываемых икринок исчисляется миллионами. Вместе с тем, имеются рыбы, которые мечут несколько сотен или десятков икринок. Объясните, почему в природе существуют те и другие рыбы.

    Элементы ответа:
    1. Самки рыб, как правило, выметывают большое количество икры в воду и она там оплодотворяется. Оплодотворение внешнее.

    2. Приспособленность к выживанию при внешнем оплодотворении — большое количество икры.

    3. У рыб с небольшой плодовитостью хорошо развита забота о потомстве, иначе они не могли бы существовать.

    Примечание: Подобные задания вызывают сложности. Критерий № 1 трудно вывести из вопроса, поскольку у подавляющего большинства рыб наружное оплодотворение. Если экзаменуемый ответит, что часть икры не оплодотворяется, выбрасывается волнами на берег или поедается другими рыбами, это будет в большей степени соответствовать смыслу вопроса. Нужно познакомить учеников с разными вариантами ответа.

    Пример 4

    Считается, что на склонах холмов поля надо распахивать поперек склона (горизонтально, террасами), а не вдоль (от вершины к подножию). Объясните, почему необходимо делать именно так и к чему может привести распашка полей вдоль склона.

    Элементы ответа:
    1. При распашке вдоль склона и вода, используемая для полива, и естественные осадки будут стекать вдоль грядок к подножию холма.

    2. Эта вода будет вымывать из почвы удобрения и другие полезные вещества, ускоряя эрозию почвы.

    3. При распашке поперек склона вода будет дольше оставаться в почве и вещества будут вымываться гораздо медленнее.

    Примечание: Школьники, как правило, не знакомы с термином «террасирование». Это задание вызывает у них серьезные затруднения в силу недостатка опыта поездок в места, где такой способ распашки распространен.

    Пример 5

    Тело пингвинов покрыто очень мощным плотным слоем контурных перьев, под которыми расположен толстый слой пуховых перьев. При этом пингвины, в отличие от других птиц, меняют пуховые перья все разом, а не постепенно в течение всей жизни. Объясните, почему у пингвинов в ходе эволюции сформировались такие особенности пухового слоя перьев и как эти особенности повышают их приспособленность к условиям окружающей среды.

    Элементы ответа:
    1. Пингвины живут в холодных условиях, поэтому мощный пуховой слой (пуховые перья особого строения) им необходим в качестве термоизоляции.

    2. Если бы пуховые перья менялись постепенно, это приводило бы к нарушению плотности контурных перьев, что, в свою очередь, приводило к намоканию перьев при плавании.

    3. Поэтому у пингвинов выработалась смена всех пуховых перьев разом, чтобы период смены был как можно короче.

    Примечание: Абитуриент может не понять смысл вопроса. Нужно уделить время этому заданию, представить веер ответов. Лучше — пользуясь дополнительной информацией. Необходимо проговорить, что линька у пингвинов длится около 20 дней — это короткое время, в течение которого птицы голодают и скапливаются в стаи, чтобы согреться. Пуховые перья выталкиваются новыми перьями. Если словосочетание «все разом» будет понято абитуриентом как «очень быстро» (что вполне вероятно), возникнут трудности с ответом.

    Пример 6

    Что происходит с признаками и характеристиками организмов при дивергентном видообразовании? Какие движущие силы эволюции лежат в основе этого процесса? Какая форма естественного отбора лежит в основе этого процесса?

    Элементы ответа:
    1. При дивергенции происходит расхождение признаков.

    2. Дивергенция обусловлена наследственной изменчивостью, борьбой за существование и естественным отбором.

    3. Движущая форма естественного отбора, ведущая к полиморфизму.

    Примечание: Термин полиморфизм будет часто встречаться в заданиях 2018 года. В некоторых вопросах подобного типа нужно будет говорить о дизруптивной форме отбора.

    Пример 7

    Почему биологический регресс часто ведет к вымиранию вида? Ответ обоснуйте, приведите не менее четырех аргументов.

    Элементы ответа:
    1. При биологическом регрессе резко сокращается численность вида в связи с понижением адаптации организмов при изменении условий среды.

    2. Происходит уменьшение ареала за счет уменьшения численности.

    3. Возникает близкородственное скрещивание, которое приводит к проявлению вредных мутаций и гибели организмов.

    4. Случайные факторы повышают вероятность вымирания вида.

    Примечание: Задание требует умения оперировать понятиями «регресс», «адаптация», «ареал», «мутации». Оно несложное, однако абитуриенты не всегда могут привести именно четыре полных критерия. Возможный веер ответов: организмы не успевают приспособиться к изменяющимся условиям среды; они не выдерживают конкуренции с другими видами; в результате они отсеиваются естественным отбором; из-за снижения численности происходит сокращение ареала. Каждая из этих формулировок вполне соответствует алгоритму, приведенному в критериях.

    Вниманию учащихся и учителей предлагается новое учебное пособие, которое поможет успешно подготовиться к единому государственному экзамену по биологии. Справочник содержит весь теоретический материал по курсу биологии, необходимый для сдачи ЕГЭ. Он включает в себя все элементы содержания, проверяемые контрольно-измерительными материалами, и помогает обобщить и систематизировать знания и умения за курс средней (полной) школы. Теоретический материал изложен в краткой, доступной форме. Каждый раздел сопровождается примерами тестовых заданий, позволяющими проверить свои знания и степень подготовленности к аттестационному экзамену. Практические задания соответствуют формату ЕГЭ. В конце пособия приводятся ответы к тестам, которые помогут школьникам и абитуриентам проверить себя и восполнить имеющиеся пробелы. Пособие адресовано школьникам, абитуриентам и учителям.

    Пример 8

    Докажите, что большинство современных птиц находится в состоянии биологического прогресса (с учетом особенностей птиц).

    Элементы ответа:
    1. Большое видовое разнообразие, обусловленное разнообразием и адаптациями к своим экологическим нишам.

    2. Высокая внутривидовая численность особей, связанная со сложным поведением (заботой о потомстве, перелетами, строительством разнообразных гнезд и т.д.).

    3. Широкий ареал обитания, обусловленный теплокровностью и способностью к полету.

    Примечание: В данном примере не следует говорить о четырехкамерном сердце, двойном дыхании, клюве без зубов и прочих приспособлениях к полету, поскольку об этом не спрашивают. Теплокровность правильно упоминается как адаптациия к переживанию различных условий среды. Ответ на это задание, как и в предыдущем примере, поддается алгоритмизации: все, что сокращается при регрессе, увеличивается при прогрессе, и наоборот.

    Пример 9

    Опишите состав первичной атмосферы Земли и условия, при которых происходил абиогенный синтез первых органических веществ. Какие вещества синтезировали Миллер и Юри в своем эксперименте? Почему эти вещества не образуются в настоящее время, например при извержениях вулканов?

    Элементы ответа:
    1. Первичная атмосфера Земли содержала водяной пар, аммиак, водород и метан.

    2. Условиями для синтеза были электрические разряды и высокая температура.

    3. В результате опыта ученые получили несколько аминокислот, мочевину и молочную кислоту.

    4. Образующиеся органические вещества сразу поглощаются микроорганизмами или окисляются кислородом атмосферы.

    Примечание: Очевидно, что в ответе должно быть 4 критерия. Данный пример показывает, что необходимо знать имена некоторых ученых и их работы.

    Пример 10

    Клевер произрастает на лугу, опыляется шмелями. Какие биотические факторы могут привести к сокращению численности популяции клевера?

    Элементы ответа:
    1. Уменьшение численности шмелей.

    2. Увеличение численности растительноядных животных.

    3. Размножение растений-конкурентов (злаков и др.).

    Примечание: В задании могут быть приведены другие критерии:

    1. Вытаптывание клевера коровами.

    2. Разрушение гнезд шмелей птицами.

    3. Уничтожение яиц шмелей насекомыми-наездниками и т.д.

    Пример 11

    Какие растения в природных условиях получают минеральное питание не из почвы, и поясните — как?

    Пример 12

    Объясните, почему для возобновления вида орлана-белохвоста достаточно двух яиц в кладке, а в кладке соловья — 6-7 яиц.

    Элементы ответа:
    1. Высокой плодовитостью отличаются виды, у которых велика гибель особей в природе.

    2. У соловья избыточное производство яиц как бы покрывает их возможную гибель (гнездовой тип птенцов, много врагов, длительные перелеты на места зимовья и др.).

    3. У видов с хорошо развитой заботой о потомстве гибель птенцов невелика (более зрелые птенцы, отсутствие естественных врагов, оседлый образ жизни).

    Примечание: Задание требует от абитуриентов как минимум некоторого представления об орлане-белохвосте. Условие отчасти наводит на правильный ответ — по крайней мере, учащиеся могут сравнить размеры птиц, предположить заботу о потомстве у орланов, и, таким образом, вывести первый критерий.

    Вниманию учащихся и учителей предлагается новое учебное пособие, которое поможет успешно подготовиться к единому государственному экзамену по биологии. Сборник содержит вопросы, подобранные по разделам и темам, проверяемым на ЕГЭ, и включает задания разных типов и уровней сложности. В конце пособия приводятся ответы на все задания. Предлагаемые тематические задания помогут учителю организовать подготовку к единому государственному экзамену, а учащимся — самостоятельно проверить свои знания и готовность к сдаче выпускного экзамена. Книга адресована учащимся, учителям и методистам.

    Пример 13

    Объясните влияние плотности водной среды обитания на «живые организмы» биоценозов.

    Элементы ответа:
    1. Свет проникает в воду на небольшую глубину, поэтому растительные организмы могут существовать только в верхних слоях (до 150-200 м).

    2. Плотность водной среды оказывает влияние на обтекаемую форму тела и сильную мускулатуру быстро передвигающихся животных.

    3. Плотность среды облегчает вес организмов и создает возможность постоянно находиться в толще среды (планктон).

    4. Наличие планктона делает возможным фильтрационный тип питания многих животных.

    Примечание: Критерий № 4 едва ли имеет прямое отношение к вопросу, поскольку указано не прямое влияние воды на жизнедеятельность организмов. Возможный веер ответов: у водных растений слабо развитая механическая ткань и высокая плавучесть; у животных развиты такие приспособления, как слизь на коже, плавники, адаптация к разным глубинам и так далее. Задания по теме «Экология организмов» требуют умения применять знания в измененной ситуации. В вопросах встречается влияние света, влажности, солености и других факторов.

    Пример 14

    Укажите не менее четырех факторов водной среды обитания. Объясните их роль в жизни организмов.

    Элементы ответа:
    1. Плотность воды определяет ее выталкивающую силу (распределение организма по разным глубинам).

    2. Температурный режим более сглажен, нет чрезмерно высоких и низких температур.

    3. Ограниченное количество кислорода. Бывают заморы в водоемах по разным причинам.

    4. Солевой состав ограничивает распространение пресноводных и морских обитателей.

    Примечание: Подобные типы заданий могут касаться наземно-воздушной, почвенной, внутриорганизменной сред обитания. Речь идет о физико-химических свойствах и их роли в жизни организмов. Абитуриент ответит правильно, если такие свойства им хорошо изучены.

    Экзамен по биологии относится к числу выборочных и сдавать ее будут те, кто уверен в знаниях. ЕГЭ по биологии считается сложным предметом, так как проверяются знания, накопленные за все годы изучения.

    Задания ЕГЭ по биологии подобраны разнотипные, для их решения необходимы уверенные знания по основным темам школьного курса биологии. На основе педагоги разработали свыше 10 тестовых заданий по каждой теме.

    Темы, которые нужно изучить при выполнении заданий смотрите от ФИПИ. Для каждого задания прописан свой алгоритм действий, который поможет при решении задач.

    Изменения в КИМ ЕГЭ 2019 г. по биологии:

    • Изменена модель задания в линии 2. Вместо задания с множественным выбором на 2 балла включено задание на работу с таблицей на 1 балл.
    • Максимальный первичный балл уменьшился на 1 и составил 58 баллов.

    Структура заданий ЕГЭ по биологии:

    • Часть 1 – это задания с 1 по 21 с кратким ответом, на выполнение отводится примерно до 5 минут.

    Совет : внимательно читайте формулировки вопросов.

    • Часть 2 – это задания с 22 по 28 с развернутым ответом, на выполнение отводится примерно 10-20 минут.

    Совет : литературно излагай свои мысли, отвечай на вопрос подробно и всесторонне, давай определение биологическим терминам, даже если этого не требуют в заданиях. В ответе должен быть план, не писать сплошным текстом, а выделять пункты.

    Что требуются от ученика на экзамене?

    • Умение работать с графической информацией (схемы, графики, таблицы) – ее анализ и использование;
    • Множественный выбор;
    • Установление соответствия;
    • Установление последовательности.


    Баллы за каждое задание по биологии ЕГЭ

    Для того, чтобы получить наивысшую оценку по биологии, необходимо набрать 58 первичных баллов, которые будут переведены в сто по шкале.

    • 1 балл — за 1, 2, 3, 6 задания.
    • 2 балла — 4, 5, 7-22.
    • З балла — 23-28.


    Как подготовиться к тестовым заданиям по биологии
    1. Повторение теории.
    2. Правильное распределение времени на каждое задание.
    3. Решение практических задач по несколько раз.
    4. Проверка уровня знаний путем решения тестов онлайн.

    Регистрируйся, занимайся и получай высокий балл!

    Последний школьный год — сложный и ответственный период. От результатов ЕГЭ зависит многое в дальнейшей жизни: выбор профессии, успех в карьере.

    Вопросы, которые встают перед каждым 11-классником: с помощью каких пособий и интернет-ресурсов я буду готовиться, как изучу огромный объем материалов, как я решу ЕГЭ, как добьюсь высоких баллов?

    Наша задача, моя как репетитора и ваша как учеников, сделать подготовку к ЕГЭ по биологии очень качественной и не тратить силы и время впустую. По результатам многих лет работы я создал большое количество учебных материалов, которые позволяют учить биологию быстро, глубоко, качественно. На моем Ютуб-канале выложены видео, в которых рассматриваются все основные темы школьного курса биологии.. Наконец, мной разработана уникальная система онлайн тестов , позволяющая изучить биологию на самом высоком уровне.

    Казалось бы, материалов для подготовки к ЕГЭ по биологии в интернете очень много, зачем репетитору вкладывать силы в написание статей, запись видео, разработку новых тестов, — ведь есть, например, проект Решу ЕГЭ, и можно готовиться по нему? К сожалению, задания по биологии там создаются не одним профессиональным преподавателем, а стекаются из разных источников. Я не берусь гарантировать их качество, зная, сколько тестов с ошибками гуляет по сети.

    В своих учебных материалах я работаю только с сертифицированными тестами ФИПИ! Они выверены, точны, в них нет ошибок, — а если попадаются «скользкие» формулировки, мы обязательно об этом говорим на занятиях. И потому все свои видео, статьи и тесты онлайн я с полной ответственностью рекомендую ученикам.

    Каталог заданий ЕГЭ по биологии Гущин с пояснениями — достаточно ли в нем информации?


    Я соглашусь, что каталог заданий Решу ЕГЭ удобен, потому что к тесту прилагается пояснение. Однако пояснение это очень короткое, тезисное. К тому же оно расслабляет ученика: не знаешь верный ответ — сразу читай пояснение! По моему мнению, школьнику необходимо дать гораздо более глубокие знания, показать внутренние связи, предупредить возможные ошибки.

    3. — это качественный тренажер, не позволяющий ученику расслабляться. Обучение здесь подкрепляется эмоциями: раздражение от того, что тест не дается, радость при его решении, — всё это делает процесс динамичным, а значит, гораздо более результативным.

    Анализ типичных ошибок при выполнении заданий ЕГЭ по биологии 2019 года

    1. Анализ типичных ошибок при выполнении заданий ЕГЭ по биологии 2019 года

    Вариант КИМ ЕГЭ экзаменационной работы содержит 28 заданий и состоит
    из двух частей, различающихся по форме и уровню сложности.
    Часть 1 содержит 21 задание: 6 – с множественным выбором с рисунком и
    без рисунка; 1 – на установление соответствия с рисунком или без рисунка;
    3 – на установление последовательности систематических таксонов,
    биологических объектов, процессов, явлений;
    2 – на решение биологических задач по цитологии и генетике;
    1 – на дополнение недостающей информации в схеме;
    2 – на дополнение недостающей информации в таблице;
    1 – на анализ информации, представленной в графической или табличной
    форме.

    2. Анализ типичных ошибок при выполнении заданий ЕГЭ по биологии 2019 года

    Часть 1 содержала задания двух уровней сложности: 12
    заданий базового уровня и 9 заданий повышенного уровня.
    ЛИНИЯ 1. Биологические термины и понятия. Дополнение схемы.
    СРЕДНИЙ % — 66%.
    ЛИНИЯ 2 предлагает задание на заполнение таблицы. СРЕДНИЙ %
    — 55%
    Пример — Рассмотрите таблицу «Методы биологических исследований».
    Запишите в ответе пропущенный термин, обозначенный в таблице
    вопросительным знаком.

    3. Анализ типичных ошибок при выполнении заданий ЕГЭ по биологии 2019 года

    Линия 3 традиционно представлена цитологической задачей.
    Средний % — 75%
    Линия 6 — задача на родословные. Справилась большая часть
    обучающихся. Треть учеников со слабой подготовкой выполнили это
    задание. Средний % — 60%
    Линия 4. Проверяются знания клеточного строения организмов.
    Пример — представлен рисунок бактериальной клетки. Необходимо
    осуществить множественный выбор. Задание оценивается 2 баллами.
    Средний % выполнения — 56%
    Линия 5. Клетка как биологическая система. Строение клетки,
    метаболизм. Средний % — 55,5%. Повышенный уровень сложности.
    2 балла.

    4. Анализ типичных ошибок при выполнении заданий ЕГЭ по биологии 2019 года

    Линия 7. Организм как биологическая система. Селекция.
    Биотехнология. Множественный выбор (без рисунка). Средний % 19,5%
    Группа с отличной подготовкой справилась с заданием, а с хорошей
    подготовкой обучающиеся выполнили задание частично и получили
    1 балл — 56%. Пример Все приведённые ниже термины, кроме двух, используют для
    описания двойного оплодотворения у покрытосеменных. Определите
    два термина, «выпадающих» из общего списка, и запишите в
    таблицу цифры, под которыми они указаны.
    1) зигота, 2) триплоидная клетка, 3) опыление, 4) мейоз, 5)
    зародышевый мешок

    5. Анализ типичных ошибок при выполнении заданий ЕГЭ по биологии 2019 года

    Линия 8. Организм как биологическая система. Установление
    соответствия (без рисунка или с рисунком). Средний% — 33,5%.
    Только 50% учащихся с хорошей и отличной подготовкой
    справились с заданием. 2 балла. Пример — формы полового и
    бесполого размножения у представителей животных, растений и
    грибов.
    Линия 9. Многообразие организмов. Растения. Множественный
    выбор (с рисунком или без рисунка). Средний % — 53%.
    Пример — проверяются знания циклов развития растений (развитие
    и размножение покрытосеменных растений) и умение
    осуществлять множественный выбор.

    6. Анализ типичных ошибок при выполнении заданий ЕГЭ по биологии 2019 года

    Линия 10. Многообразие организмов. Растения. Установление
    соответствия (с рисунком и без рисунка).Средний % — 0% (9,7%
    — 1 балл)
    Пример — Установите соответствие между структурами и группами
    тканей: к каждой позиции, данной в первом столбце, подберите
    соответствующую позицию из второго столбца.
    СТРУКТУРЫ
    ГРУППЫ ТКАНЕЙ
    А) устьице
    1) проводящие
    Б) механическое волокно
    2) покровные
    В) пробка
    Г) корневой волосок
    Д) ситовидная трубка
    Е) железистый волосок

    7. Анализ типичных ошибок при выполнении заданий ЕГЭ по биологии 2019 года

    Линия 11. Многообразие организмов. Основные систематические
    категории, их соподчинённость Установление последовательности
    Средний % — 67,5%.
    Линия 12. Организм человека. Гигиена человека. Множественный
    выбор (с рисунком и без рисунка). Средний % — 25,3%.
    Пример — Выберите три верных ответа из шести и запишите в таблицу
    цифры, под которыми они указаны.
    К болезням, вызванным пониженной функцией эндокринных желёз,
    относят
    1) микседему
    2) базедову болезнь
    3) гигантизм
    4) карликовость
    5) сахарный диабет
    6) акромегалию

    8. Анализ типичных ошибок при выполнении заданий ЕГЭ по биологии 2019 года

    Линия 13. Организм человека. Установление
    соответствия (пример с рисунком). Средний % — 35%
    Установите соответствие между
    функциями и отделами головного мозга
    человека, обозначенными на рисунке
    цифрами 1, 2:
    к каждой позиции, данной
    в первом столбце, подберите
    соответствующую позицию

    9. Анализ типичных ошибок при выполнении заданий ЕГЭ по биологии 2019 года

    Линия 14. Организм человека. Установление
    последовательности. 64%
    Линия 15. Эволюция живой природы. Множественный
    выбор (работа с текстом). 90,4%
    Линия 16. Эволюция живой природы. Происхождение
    человека. Установление соответствия. 32,5%
    ПРИМЕР — ПРИЗНАКИ ОРГАНИЗМОВ и ПУТИ ЭВОЛЮЦИИ
    1) ароморфоз, 2) идиоадаптация, 3) общая дегенерация

    10. Анализ типичных ошибок при выполнении заданий ЕГЭ по биологии 2019 года

    Линия 17. Экосистемы и присущие им закономерности.
    Биосфера. Множественный выбор. 50,6%. Пример стадии первичной сукцессии.
    Линия 18. Экосистемы и присущие им закономерности.
    Биосфера. Установление соответствия. 30%
    Пример —
    ОРГАНИЗМЫ
    А) медоносная пчела
    Б) стрекоза коромысло
    В) божья коровка
    Г) колорадский жук
    Д) таёжный клещ
    Е) среднеазиатская саранча
    ФУНКЦИОНАЛЬНЫЕ ГРУППЫ В ЭКОСИСТЕМЕ
    1) консумент I порядка
    2) консумент II порядка

    11. Анализ типичных ошибок при выполнении заданий ЕГЭ по биологии 2019 года

    Линия 19. Общебиологические закономерности.
    Установление последовательности (пример –
    появление покровительственной окраски у
    насекомых). 55%
    Линия 20. Задание на анализ информации и заполнение
    таблицы. 71%
    Линия 21. Биологические системы и их
    закономерности. Анализ данных, в табличной или
    графической форме. 32,5%
    Необходимо отработать сюжеты подобных заданий по
    пособиям. Научиться читать информацию.
    Часть 2 содержит 7 заданий с
    развёрнутым ответом.
    1 задание на два элемента ответа и 6 заданий на три и
    более элемента.
    В этих заданиях ответ формулируется и записывается
    экзаменуемым самостоятельно в развёрнутой форме.
    Задания этой части работы нацелены на выявление
    выпускников, имеющих высокий уровень
    биологической подготовки.
    Необходимо отметить, что все варианты,
    предложенные в регионе, содержали задания,
    примерно равнозначные по уровню сложности.

    13. Анализ типичных ошибок при выполнении заданий ЕГЭ по биологии 2019 года

    Линия 22. Применение биологических знаний в
    практических ситуациях (практико-ориентированное
    задание). 9,6%.
    Пример — Почему препарат инсулина, необходимый для
    лечения больных диабетом, выпускается только в
    виде раствора для инъекций, а не в виде таблеток?
    Линия 23. Задание с изображением биологического
    объекта. 12%
    Пример — Используя фрагмент «Геохронологической таблицы»,
    определите, в какой эре и каких периодах обитал данный
    организм. С какими современными отделами это растение имеет
    сходство и по каким признакам?

    14. Анализ типичных ошибок при выполнении заданий ЕГЭ по биологии 2019 года

    Линия 24. Задание на анализ биологической
    информации. Работа с текстом по исправлению
    ошибок. 7%. (пример –»Дыхательная система
    человека».
    Линия 25. Обобщение и применение знаний о
    человеке и многообразии организмов. 14,5%
    Линия 26. Обобщение и применение знаний об
    эволюции органического мира и экологических
    закономерностях в новой ситуации. 5%

    15. Анализ типичных ошибок при выполнении заданий ЕГЭ по биологии 2019 года

    Линия 27. Решение задач по цитологии на применение
    знаний в новой ситуации. 25%.
    Линия 28. Решение задач по генетике на применение
    знаний в новой ситуации. 20,5% Пример У бабочек гетерогаметным полом является женский пол. При скрещивании
    самки бабочки с длинными усами, однотонным окрасом крыльев и самца с
    короткими усами, наличием пятен на крыльях в потомстве получились
    самки с длинными усами, наличием пятен на крыльях и самцы с
    длинными усами, однотонным окрасом. При скрещивании самки бабочки с
    короткими усами, наличием пятен на крыльях и самца с длинными усами,
    однотонным окрасом крыльев всё гибридное потомство было
    единообразным по длине усов и окраске крыльев. Составьте схемы
    скрещиваний.
    Определите генотипы родительских особей, генотипы и фенотипы
    потомства в двух скрещиваниях, пол потомства в каждом скрещивании.
    Объясните фенотипическое расщепление в первом скрещивании.

    16. Модель КИМов ЕГЭ 2020 года

    Изменений в структуре КИМов в 2020
    году нет.
    Есть усложнения заданий линии 27.
    Цитологические задачи.

    17. Модель КИМов ЕГЭ 2020 года

    Пример задания 2019 года.
    Фрагмент начала гена имеет следующую последовательность
    нуклеотидов: ГАТАЦТТАТГАЦТАГААТЦА
    Ген содержит информативную и неинформативную части для
    трансляции. Информативная часть гена начинается с
    триплета, кодирующего аминокислоту Мет.
    С какого нуклеотида начинается информативная часть гена?
    Определите последовательность аминокислот во фрагменте
    полипептидной цепи. Ответ поясните.
    Для выполнения задания используйте таблицу генетического
    кода.
    Пример из демоверсии 2020 года.
    Известно, что все виды РНК синтезируются на ДНК-матрице.
    Фрагмент молекулы ДНК, на которой синтезируется участок
    центральной петли тРНК, имеет следующую
    последовательность нуклеотидов (верхняя цепь смысловая,
    нижняя транскрибируемая).
    5’-ЦГААГГТГАЦААТГТ-3’
    3’-ГЦТТЦЦАЦТГТТАЦА-5’
    Установите нуклеотидную последовательность участка тРНК,
    который синтезируется на данном фрагменте, обозначьте 5’ и 3’
    концы этого фрагмента и определите аминокислоту, которую
    будет переносить эта тРНК в процессе биосинтеза белка, если
    третий триплет с 5’ конца соответствует антикодону тРНК.
    Ответ поясните. Для решения задания используйте таблицу
    генетического кода.
    Спасибо
    за внимание!

    На Луну: определение и обнаружение накачки криптовалюты | Crime Science

    Криптовалюты привлекают все большее внимание общественности, и их использование в качестве инвестиционной платформы растет. Эти цифровые валюты упрощают платежи в онлайн-секторе без необходимости в центральном органе (например, в банке). Рынок криптовалют быстро расширяется, и на момент написания статьи его рыночная капитализация составляла около 300 миллиардов долларов США (CoinMarketCap 2018), что делает его сопоставимым с ВВП Дании (Cryptocurrency Prices 2018).Несмотря на то, что в криптовалюты инвестируются и продаются огромные суммы денег, они остаются неизведанной территорией и по большей части не регулируются. Отсутствие регулирования в сочетании с их технической сложностью делает их привлекательной мишенью для мошенников, которые будут охотиться на дезинформированных. Одна из таких афер известна как «накачка и сброс» (P&D), когда злоумышленники пытаются получить прибыль, распространяя дезинформацию о товаре (то есть о конкретной криптовалютной монете), чтобы искусственно поднять цену (Kramer 2004).Эта афера имеет долгую историю в традиционных экономических условиях, начиная с лондонской компании Южных морей в 1700-х годах (Brooker 1998), затем нашла естественный приют в копеечных ценных бумагах и в Интернете (Kramer 2004; Temple 2000) и теперь недавно появился на рынках криптовалют (Хан 2018; Мак и Литвиненко 2018; Мартино 2018).

    Академическая литература по криптовалютным схемам P&D скудна (за исключением см. Недавний рабочий документ Li, Shin, & Wang, 2018).Таким образом, эта статья даст обзор того, что в настоящее время известно по этой теме из блогов и новостных сайтов. Для получения теоретической точки зрения изучается экономическая литература, связанная с этой темой, и эта информация синтезируется с криптовалютами, выделяя сходства и потенциальные различия. Поскольку эти закономерности являются разновидностью аномалии, также обсуждается литература по алгоритмам обнаружения аномалий. Цель состоит в том, чтобы предложить некоторые определяющие критерии того, что такое крипто-P&D, и впоследствии использовать эту информацию для обнаружения точек в обменных данных, которые соответствуют этим критериям, формируя основу для дальнейших исследований.

    Что такое схема откачки и откачки?

    Схема накачки и сброса — это вид мошенничества, при котором преступники накапливают товар в течение определенного периода, а затем искусственно завышают цену с помощью распространения дезинформации (накачки), прежде чем продать то, что они купили ничего не подозревающим покупателям на месте. более высокая цена (демпинг). Поскольку цена была завышена искусственно, цена обычно падает, оставляя покупателей, купивших на основании ложной информации, в убытке. Хотя мы не проводим тщательного анализа сценария преступления (см. Borrion 2013; Keatley 2018; Warren et al.2017) здесь, рис.1 можно рассматривать как абстракцию скрипта, состоящую из трех основных этапов — накопления, накачки и сброса. Фаза накопления обычно происходит постепенно в течение более длительного периода времени, чтобы избежать повышения цены перед накачкой.

    Рис. 1

    Схематическая абстракция трех фаз операции накачки и сброса

    Что такое криптовалюты?

    Криптовалюты — это цифровое средство обмена, и для предотвращения таких проблем, как подделка, они обычно полагаются на криптографию, а не на центральное учреждение.Например, самой популярной криптовалютой является биткойн (BTC), и некоторые из ее преимуществ заключаются в том, что она позволяет проводить безнадежные и децентрализованные транзакции, поскольку невозможно отменить платеж, и в нем не участвуют третьи стороны (например, банки). (Накамото 2008). В традиционных финансовых системах клиент доверяет третьей стороне (например, банку) обновление своей бухгалтерской книги, чтобы отразить баланс счетов клиента. Напротив, в Биткойне этот реестр распределен по сети, и каждый в сети имеет его копию и может — в принципе — проверять ее содержимое.Этот публичный реестр известен как блокчейн и является базовой технологией, на которой основывается Биткойн и многие другие криптовалюты. В настоящее время существует множество различных типов криптовалют, менее известные из которых называются «альткойнами», и все они работают на несколько разных технических принципах, с разными утилитами и преимуществами (Bitcoin Magazine 2017). Помимо Биткойна, некоторые из других наиболее популярных в настоящее время криптовалют включают Ethereum (https://ethereum.org/), Ripple (https: // litecoin.org /) и Litecoin (https://litecoin.org/).

    Цели этого документа

    В этом документе мы поставили перед собой задачу достичь трех основных целей. Во-первых, в отсутствие академических исследований схем накачки и сброса криптовалюты, мы предоставили начальную рабочую формализацию крипто-P&D, определяющих критерии, которые могут помочь в обнаружении и идеальном предотвращении этой возникающей проблемы мошенничества. Во-вторых, мы используем эти индикаторы и предлагаем автоматизированный подход к обнаружению аномалий для выявления моделей подозрительных транзакций.В-третьих, чтобы лучше понять феномен P&D криптовалюты, мы увеличиваем масштаб обмена и уровня пар криптовалют. Основная цель данной статьи — вызвать интерес ученых к этой теме и представить P&D как возникающую проблему.

    Схемы откачки и сброса в традиционном экономическом контексте

    В начале восемнадцатого века аферисты, владевшие акциями компании Южного моря, начали делать ложные заявления о компании и ее прибылях. Цель заключалась в том, чтобы искусственно поднять цену акций, а затем продать их дезинформированным покупателям, которые были убеждены, что покупают многообещающий товар.Это было названо Южным морским пузырем и служит ранним задокументированным примером схемы P&D (Bartels 2000; Brooker 1998).

    В наше время схемы P&D в основном основаны на Интернете с упором на так называемые «пенни» или «микрокапитальные» акции, то есть более мелкие компании, не соответствующие требованиям для листинга на более крупных биржах, таких как NASDAQ ( Dugan 2002; Temple 2000). Биржи микрокапиталов не придерживаются одного и того же стандарта регулирования, что означает, что обычно не так много информации о компаниях, котирующихся на бирже, что упрощает манипулирование ими.Например, в США крупные публичные компании подают общедоступные отчеты в Комиссию по ценным бумагам и биржам (SEC), которые часто анализируются профессионалами (Комиссия по ценным бумагам и биржам США, 2017). Доступ к информации и ее проверка обычно более трудны для компаний с микрокапиталом. Дезинформация об акциях часто распространяется через спам по электронной почте, который, как было установлено, оказывает чистое положительное влияние на цену акций (то есть спам эффективно увеличивает цену, см. Bouraoui 2009).В Соединенных Штатах незаконно проводить операции P&D с мелкими акциями, и есть несколько случаев, когда против людей предъявлялись обвинения за участие в мошенничестве с P&D («События в банковском и финансовом законодательстве: 2013, 2014; Ян и Уорден 2015).

    Схемы накачки и сброса в контексте криптовалюты

    В настоящее время не хватает академической литературы по схемам накачки и сброса криптовалюты, поэтому в этом разделе делается попытка дать обзор текущего ландшафта схем P&D криптовалюты в том виде, в каком они есть реализовано в различных блогах и новостных статьях.В контексте криптовалюты в целом существует несколько иной способ работы, чем в традиционном контексте пенсовых акций; в частности, это проявилось в росте числа специализированных общественных групп P&D. Эти группы появились в онлайн-чатах, таких как Discord (https://discordapp.com) и Telegram (https://telegram.org), с единственной целью — организовывать мошенничество с накачкой и сбросом некоторых криптовалют (рис. ). Сообщается, что количество членов в некоторых из этих групп достигло 200000 человек, а в небольших группах все еще работает около 2000 человек (Martineau, 2018).Наблюдается рост цен до 950%, что свидетельствует о масштабах манипуляций, на которые способны эти группы (Thompson 2018). Чтобы эти группы P&D достигли наилучших результатов, несколько отчетов об активности показывают, что они почти исключительно нацелены на менее популярные монеты, особенно с низкой рыночной капитализацией и низким тиражом, поскольку ими легче манипулировать (Хан, 2018; Мак и Литвиненко, 2018). ; Город 2018). Оценить полный объем ущерба, причиненного накачкой и сбросом криптовалюты, сложно; тем не менее, есть некоторые свидетельства того, что такие схемы приносят миллионы долларов торговой деятельности.Wall Street Journal опубликовал статью-расследование, в которой анализировались публичные группы памп-и-сброса и 6 месяцев торговой деятельности. Они обнаружили, что 825 миллионов долларов связаны со схемами откачки и сброса, при этом только на одну группу приходится 222 миллиона долларов в сделках (Shifflett, 2018). Это дает представление о том, какая денежная активность создается этими группами, влияние которой может быть еще большим, поскольку многие группы предположительно действуют в частных группах или группах только по приглашению.

    Рис. 2

    Пример чат-группы с накачкой и сбросом с более чем 40 000 участников.Слева: Telegram-группа «Ракетная свалка». Справа: соответствующие данные обмена (Binance) целевой монеты (Yoyo), показывающие эффект накачки. Желтая, пурпурная и бордовая линии представляют собой скользящую среднюю за последние 7, 25 и 99 дней соответственно. конкретный обмен, и только по истечении указанного времени монета будет объявлена ​​(см. рис. 2). После объявления монеты участники группового чата стараются быть одними из первых, кто купит монету, чтобы получить больше прибыли.В самом деле, если они будут слишком медленными, они могут в конечном итоге купить на пике и не смогут продать с прибылью. «Шумиха» вокруг покупки монеты после объявления о помпе объясняется короткими временными рамками этих схем: Мартино (2018) сообщил о двух помпах, которые достигли своего пика в течение 5–10 минут. Во время фазы накачки пользователей часто поощряют распространять дезинформацию о монете, пытаясь обманом заставить других купить ее, что позволяет им легче продавать. Дезинформация варьируется, но некоторые распространенные тактики включают ложные новости, несуществующие проекты, поддельные партнерские отношения или поддельные одобрения знаменитостей (Martineau 2018; Town 2018).Рассмотрим пример, когда группа преступников выдавала себя за учетную запись в Твиттере интернет-предпринимателя Джона Макэфи. @OfficialMcafee добавив дополнительный » л ’В имени пользователя (Mac и Литвиненко, 2018). Поддельный аккаунт отправил положительный твит о конкретном альткойне, и всем пользователям группы P&D было предложено ретвитнуть его. В течение 5 мин. Цена монеты упала с 30 до 45 долларов и упала обратно до 30 долларов примерно через 20 минут.Все, что создает общий позитивный настрой, является честной игрой, потому что цель состоит в том, чтобы сбросить свои монеты на ничего не подозревающих инвесторов, которые не проявили должной осмотрительности, используя свой страх упустить следующую крупную криптовалюту.

    Стремясь обеспечить себе прибыль, многие лидеры групп накачки и сброса часто используют свою инсайдерскую информацию в своих интересах: поскольку они знают, какая монета будет накачана, они могут предварительно купить монету по более низкой цене, прежде чем они объявляют это.Это гарантирует им прибыль, в то время как другим пользователям остается по существу рисковать тем, смогут ли они предсказать пик. Страх упустить возможность и возможность превзойти шансы могут побудить потенциальных инвесторов в криптовалюту присоединиться к насосу. Руководители группы также могут гарантировать прибыль, предлагая доступ к уведомлению о насосе на более ранней стадии до объявления для всей группы в обмен на оплату. Даже нескольких секунд временного преимущества достаточно, чтобы потенциально разместить заказы на покупку раньше других и, таким образом, получить более дешевые монеты, тем самым увеличивая выгоду покупателя от операции пампа и сброса (Мартино, 2018).

    В связи с тем, что технология, лежащая в основе криптовалют, является относительно новой, а большинство бирж не регулируются, манипуляции с накачкой и сбросом в настоящее время не всегда незаконны; и даже там, где это есть, его не всегда легко добиться. Однако руководящие органы начинают осознавать эту проблему, и в Соединенных Штатах Комиссия по торговле товарными фьючерсами выпустила инструкции о том, как избежать мошенничества с P&D, а также предложила программу разоблачителей (Комиссия по торговле товарными фьючерсами США 2018).

    Определение схемы накачки и сброса криптовалюты

    Смягчение и предотвращение схем накачки и сброса потребует знания об их работе, и, таким образом, обнаружение этих схем накачки и сброса является шагом к достижению цели смягчения. Чтобы начать поиск и выявление потенциальных шаблонов типов P&D в данных обмена, необходимо рабочее определение того, что составляет P&D. Предложение по определению критериев будет дано в этом разделе путем обобщения идей относительно традиционных и криптографических схем P&D, которые были изложены в предыдущем разделе.В таблице 1 приведены некоторые ключевые сходства и различия в отношении цели, тактики и шкалы времени традиционных копеечных акций и криптографических схем накачки и сброса.

    Таблица 1 Сравнение традиционных и криптографических схем пампа и сброса

    Таблица 1 показывает, что криптовалютный P&D кажется похожим на P&D копейки в том смысле, что нацелены на активы с одинаковыми свойствами. Однако в целом кажется, что в результате применения различных тактик шкала времени сузилась и сместилась в сторону, близкую к реальному времени.Подобно тому, как оцифровка информации через Интернет увеличила уровень мошенничества P&D в отношении пенсовых акций, похоже, что оцифровка самой валюты увеличила скорость и скорость, с которой может происходить P&D.

    Использование выявленных характеристик крипто-P&D позволяет нам сформулировать критерии, которые могут быть полезны при обнаружении паттернов P&D в обменных данных (таблица 2). В частности, мы утверждаем, что индикаторы P&D могут быть подразделены на индикаторов прорыва , которые относятся к сигналам, которые всегда будут присутствовать во время накачки и сброса, и подкрепляющих индикаторов , которые относятся к индикаторам, которые могут помочь повысить уверенность в том, что наблюдаемая точка данных является результатом манипуляции.Объем и цена обсуждаются с окном оценки , относящимся к набору предыдущих точек данных некоторой заданной пользователем длины. Например, можно использовать скользящее среднее за ранее определенный период времени, что позволит обсуждать всплески в отношении некоторой местной истории. Это не означает, что предложенные критерии достаточны для охвата всех крипто-P&D. Вместо этого мы решили прибегнуть к консервативным критериям, которые необходимы для P&D и которые, судя по всему, появились на основе информации, приведенной в предыдущем разделе.

    Таблица 2 Показатели откачки-откачки по временному измерению и типу показателя

    Динамическая структурная биология на основе FRET: проблемы, перспективы и призыв к практикам открытой науки

    Понимание того, как биомолекулы соединяют структурную динамику с функцией, лежит в основе нескольких дисциплин и остается выдающейся целью биологии. Связывание конформационных состояний и их переходов с биохимической функцией требует способности точно определять структуру и динамику биологической системы, которая часто изменяется при связывании лиганда или под влиянием химических и физических свойств окружающей среды.Наиболее хорошо зарекомендовавшие себя инструменты структурной биологии предоставили с высоким разрешением «снимки» состояний в кристаллизованной или замороженной форме (например, рентгеновская кристаллография и криоэлектронная микроскопия одиночных частиц, криоЭМ) или усредненное по ансамблю всех конформаций. (например, ядерный магнитный резонанс, ЯМР; малоугловое рассеяние рентгеновских лучей, МУРР; малоугловое рассеяние нейтронов, МУРН; двойной электрон-электронный резонанс, DEER; сшивающая масс-спектрометрия, XL-MS; ансамбль-FRET). В последние годы дальнейшие разработки позволили этим традиционным структурным инструментам обнаруживать конформационную динамику и промежуточные продукты реакции.Например, методы ЯМР (Anthis and Clore, 2015; Clore and Iwahara, 2009; Palmer, 2004; Ravera et al., 2014; Sekhar and Kay, 2019) и методы электронного парамагнитного резонанса (Jeschke, 2018; Jeschke, 2012; Krstić et al., 2011) были продвинуты для изучения конформационной динамики и захвата временных промежуточных соединений. Кристаллографические исследования с временным разрешением использовались для определения функционально значимых структурных смещений, связанных с биологической функцией (Kupitz et al., 2014; Moffat, 2001; Schlichting et al., 1990; Шлихтинг и Чу, 2000; Schotte et al., 2003). Достижения в микрожидкостных устройствах для смешивания и распыления позволили использовать криоЭМ с временным разрешением (Feng et al., 2017; Kaledhonkar et al., 2018) и масс-спектрометрию с поперечными связями (XL-MS или CL-MS) (Braitbard et al., 2019 ; Brodie et al., 2019; Chen et al., 2020; Iacobucci et al., 2019; Murakami et al., 2013; Славин, Калисман, 2018). Прогресс в вычислительных методах также предоставил новые инструменты для изучения биомолекулярной структуры и динамики. Каждое из этих достижений подчеркивает возросшее понимание того, что необходимо напрямую и непрерывно отслеживать динамические свойства отдельных биомолекул, чтобы понять их функцию и регуляцию.

    В этом контексте FRET (называемый резонансным переносом энергии флуоресценции или резонансным переносом энергии Фёрстера [Braslavsky et al., 2008]) исследования на ансамблевом и одномолекулярном уровнях стали важными инструментами для измерения структурной динамики по крайней мере на протяжении 12 порядков величины во времени и картирование конформационных и функциональных неоднородностей биомолекул в условиях окружающей среды. FRET изучает затухание флуоресценции на уровне ансамбля (Grinvald et al., 1972; Haas et al., 1975; Хаас и Стейнберг, 1984; Hochstrasser et al., 1992) (FRET с временным разрешением) позволили уже в начале 1970-х годов изучать структурные неоднородности во временных масштабах, превышающих время жизни флуоресценции (несколько нс). Этот подход используется до сих пор (Becker, 2019; Orevi et al., 2014; Peulen et al., 2017) и перенесен в исследования одиночных молекул. Возможность измерения FRET в отдельных молекулах (Deniz et al., 1999; Ha et al., 1996; Lerner et al., 2018a) сделала этот метод еще более привлекательным.Одномолекулярный FRET (smFRET) широко используется для изучения конформационной динамики и биомолекулярных взаимодействий в стационарных условиях (Dupuis et al., 2014; Larsen et al., 2019; Lerner et al., 2018a; Lipman et al. ., 2003; Margittai et al., 2003; Mazal, Haran, 2019; Michalet et al., 2006; Orevi et al., 2014; Ray et al., 2019; Sasmal et al., 2016; Schuler et al., 2005; Schuler et al., 2002; Steiner et al., 2008; Zhuang et al., 2000). Примечательно, что во многих механистических исследованиях достаточно использовать FRET для различения различных конформаций и определения кинетических скоростей, так что абсолютные эффективности FRET и, следовательно, расстояния не нужно определять.Однако возможность точного измерения расстояний и кинетики с помощью smFRET привела к его появлению в качестве важного инструмента в эту новую эру « динамической структурной биологии » для картирования биомолекулярных неоднородностей и для измерения структурной динамики в широком диапазоне временных масштабов (Lerner et al., 2018a; Mazal, Haran, 2019; Sanabria et al., 2020; Schuler, Hofmann, 2013; Weiss, 1999).

    Одномолекулярный FRET (smFRET) имеет много преимуществ в качестве метода структурной биологии, в том числе:

    • чувствительность к макромолекулярным расстояниям (2.5–10 нм),

    • способность разрешать структурные и динамические неоднородности,

    • высококачественные измерения с низким потреблением образцов интересующих молекул (низкие концентрации и низкие объемы), поскольку образец анализируется по одной молекуле за раз,

    • определение структурных переходов в состоянии равновесия, следовательно, без необходимости синхронизации,

    • возможность обнаруживать (очень) редкие события.В самом деле, в биологии наиболее интересными для изучения молекулы часто являются редкие, функционально активные молекулы среди моря неактивных молекул,

    • высокая чувствительность и специфичность для меченых молекул. Поскольку только меченая молекула вносит уникальный вклад в детектируемый сигнал, эти индикаторы также могут применяться в качестве FRET-репортеров в тесноте (Dupuis et al., 2014; Soranno et al., 2014; Zosel et al., 2020b) (отсюда smFRET может использоваться для проверки результатов, определенных изолированно, или обнаружения модуляции конформационных предпочтений и / или структурной динамики посредством так называемых пяти взаимодействий [Guin and Gruebele, 2019]), и

    • высокая специфичность для остатков / доменов за счет специфической маркировки.Биомолекулы могут быть специально помечены уникальной парой красителей, что позволяет проводить измерения smFRET для всех размеров молекул, включая большие сложные сборки (см. Рисунок 1 [Kilic et al., 2018]), активные биологические машины (например, рибосомы) ( Dunkle et al., 2011) и даже на целых нативных вирионах (Lu et al., 2019; Munro et al., 2014).

    Рабочий процесс моделирования динамических структур по измерениям FRET.

    ( A ) Интеграционное моделирование требует структурной и динамической информации. Предварительная информация из традиционных подходов (рентген, ЯМР, криоЭМ) вместе с вычислительными инструментами определяет пространство возможных решений для структурного моделирования с помощью FRET. Комбинация структурной (расстояния между красителями) и динамической информации (кинетическая связь и обменные курсы) позволяет идентифицировать непротиворечивую модель. ( B ) Изучение структуры и динамики хроматиновых волокон.Комбинированное TIRF и конфокальное FRET исследование структуры и динамики хроматиновых волокон с использованием трех позиций маркировки FRET (DA1-3) для двух пар красителей с различными расстояниями Ферстера. Расстояния Фёрстера (определены в разделе Расстояния между красителями, уравнение 6). Предварительная структурная информация, полученная с помощью криоэлектронной микроскопии (вверху слева) (Song et al., 2014) и рентгеновской кристаллографии (вверху, справа PDB ID: 1ZBB Schalch et al., 2005), объединена со структурной и динамической информацией. полученные в результате экспериментов FRET на иммобилизованных молекулах, измеренных с помощью микроскопии полного внутреннего отражения (TIRF), и на свободно диффундирующих молекулах с помощью конфокальной микроскопии (Kilic et al., 2018). На основе объединенной информации получена согласованная модель конформаций хроматиновых волокон со смещенными регистрами, которые связаны медленными (> 100 мс) и быстрыми процессами декомпакции (150 мкс), которые протекают не напрямую, а через открытое волокно. конформация. Рисунок 1B был воспроизведен с рисунков 1, 3 и 6 в Kilic et al., 2018, Nature Communications с разрешения, опубликованном под Международной общественной лицензией Creative Commons Attribution 4.0 (CC BY 4.0; https: // creativecommons.org / licenses / by / 4.0 /).

    © 2018, Kilic et al. Панель B была воспроизведена с рисунков 1, 3 и 6 в Kilic et al., 2018 с разрешения, опубликованного в соответствии с Международной общественной лицензией Creative Commons Attribution 4.0.

    Несколько методов были использованы для определения структурных ансамблей, таких как ЯМР, одночастичная криоЭМ или XL-MS, а недавно также smFRET в интегративном / гибридном (I / H) подходе с компьютерным моделированием для преодоления разреженности экспериментальных данных. относительно атомистического описания (Берман и др., 2019; де Соуза и Пикотти, 2020; Димура и др., 2020; Gauto et al., 2019; Кукос и Бонвин, 2020; На и Пэк, 2020; Тан и Гонг, 2020; Webb et al., 2018). Структурные модели I / H, полученные из экспериментов smFRET с использованием расстояний между красителями в качестве ограничений, были описаны для гибких свернутых белков (Brunger et al., 2011; Hellenkamp et al., 2017; Margittai et al., 2003; McCann et al., 2012). ), конформационные ансамбли неупорядоченных / неструктурированных и развернутых белков (Borgia et al., 2018; Holmstrom et al., 2018; Schuler et al., 2020), нуклеиновые кислоты и комплексы белок-нуклеиновая кислота (Craggs et al., 2019; Craggs, Kapanidis, 2012; Kalinin et al., 2012; Lerner et al., 2018b; Muschielok et al., 2008). ; Возняк и др., 2008).

    Еще одним уникальным аспектом исследований smFRET является то, что структурная, кинетическая и спектроскопическая информация о больших и сложных системах может быть записана одновременно в одном измерении. Это облегчает объединение динамической и структурной информации в интегративный подход к (рис. 1A) (Hellenkamp et al., 2017; Килич и др., 2018; Ли и др., 2020b; Санабрия и др., 2020; Вассерман и др., 2016; Янез Ороско и др., 2018):

    .
    • определяют количество возможных структур, согласующихся с данными,

    • потенциально снижает неоднозначность между различными структурными моделями, совместимыми с экспериментальными данными, а

    • раскрывают структурно разрешенные динамические пути обмена.

    В качестве примера на рисунке 1B показан результат мультимодального исследования smFRET конформационного ландшафта 12-мерного массива хроматина (~ 2.5 MDa) (Kilic et al., 2018) с динамикой, происходящей во временных масштабах от наносекунд до часов. SmFRET эксперименты могут обнаруживать гибкие конформации хроматина (рис. 1B, средняя панель), показывая их динамическую структурную неоднородность (рис. 1B, нижняя панель), в отличие от хорошо упорядоченных статических структур хроматиновых волокон (рис. 1B, верхняя панель). Эти гибкие, частично открытые и открытые конформации, которые довольно многочисленны в растворе (популяция> 70%; Рисунок 1B, нижняя панель), не были разрешены ранее, хотя они необходимы для правильной организации и функции гена.Они представляют собой центральный узел взаимопревращений для отдельных регистров стэкинга хроматина и их трудно обнаружить с помощью других структурных методов. Такой подход к визуализации биомолекул в действии в условиях окружающей среды подчеркивает важность их динамической природы, разрешая переходы между различными конформационными состояниями, что во многих случаях способствует их функции (Aviram et al., 2018; Henzler-Wildman et al., 2007; Iljina et al., 2020; Lerner et al., 2018b; Sanabria et al., 2020; Tassis et al., 2020).

    Измерения

    SmFRET обычно выполняются с использованием двух подходов: с использованием иммобилизованных на поверхности молекул с использованием флуоресцентной микроскопии полного внутреннего отражения (TIRFM) и обнаружения на основе камеры или со свободно диффундирующими молекулами в растворе с использованием конфокальной микроскопии и точечных детекторов. Экспериментальные системы доступны в продаже, но, как правило, их изготавливают самостоятельно. Образцы готовятся, а данные собираются с использованием специальных лабораторных протоколов, где данные хранятся в различных форматах файлов и анализируются с использованием набора все более мощного программного обеспечения.Для полевых исследований в целом и для структурных исследований в частности важно продемонстрировать, что smFRET как метод воспроизводим и надежен независимо от того, где и как измеряется образец. С этой целью под руководством Торстена Хугеля двадцать лабораторий объединились для измерения smFRET на нескольких конструкциях дцДНК (Hellenkamp et al., 2018a). Изучая шесть различных образцов с разными красителями и различными расстояниями между красителями, средняя эффективность FRET, полученная участвующими лабораториями, показала удивительно высокую степень согласия (ΔE между 0.02 и 0,05 в зависимости от деталей образца). Количественная оценка и воспроизводимость измерений smFRET на основе интенсивности и обсуждение анализа данных стали важной вехой. Эти стандарты дцДНК FRET теперь доступны для ежедневной калибровки и особенно полезны для новых групп, присоединяющихся к сообществу.

    Вдохновленный идеями, полученными в ходе вышеупомянутой попытки FRET (Hellenkamp et al., 2018a), были начаты новые многолабораторные слепые исследования.Следующее сравнительное исследование FRET, проведенное Thorben Cordes, исследует надежность и надежность экспериментов smFRET на белках, претерпевающих индуцированные лигандом конформационные изменения (Gebhardt et al., В стадии подготовки). В этом исследовании используются два различных модельных белка для оценки воспроизводимости и точности smFRET на основе белков для измерения расстояния между красителями. Белковые системы ставят новые задачи, включая статистическую маркировку красителей, специфичные для сайта свойства красителей, стабильность белков, транспортировку, хранение и конформационную динамику.Следовательно, в исследовании также оценивается способность smFRET обнаруживать и количественно оценивать динамику в различных временных масштабах от микросекунд до секунд. Еще одна задача FRET, инициированная Соней Шмид, — это программа kinSoftChallenge (http://www.kinsoftchallenge.com, Götz et al., В стадии подготовки), которая оценивает существующие инструменты для извлечения кинетической информации из временных траекторий одиночных молекул. Эта задача направлена ​​на: (1) продемонстрировать способность кинетического анализа на основе smFRET точно выводить динамическую информацию и (2) предоставить сообществу средства оценки различных доступных программных инструментов.

    Одним из важных результатов различных исследований FRET в нескольких лабораториях было то, что, хотя согласие было хорошим, его можно было улучшить еще больше. В частности, анализ данных и, в частности, исправления могут повлиять на определенную эффективность FRET и результирующие расстояния. Следовательно, открытое обсуждение того, какие подходы работают наиболее надежно, при каких условиях необходимо. Доступ к первичным данным и возможность их обработки с помощью различных подходов к анализу были и останутся наиболее прозрачным способом продвижения вперед в этой области.В настоящее время это сложно, учитывая множество вариантов используемых методов, их документации, форматов файлов и экспериментальных процедур, применяемых в разных лабораториях, для установления оптимальных условий, рабочего процесса и передовых практик даже для существующих, хорошо протестированных методов, поскольку сравнение этих методов затруднительно. требует много времени, а необходимая информация во многих случаях недоступна. С расширением открытых научных практик и представлением опубликованных данных в репозитории необходим консенсус относительно того, какие данные и метаданные следует хранить и в каких возможных форматах, чтобы их могло легко использовать сообщество.

    В связи с этими соображениями и множеством возможностей для роста сообщества smFRET, несколько лабораторий, обладающих опытом в FRET, без претензии на исчерпывающий или исключительный характер, собрались, чтобы поддержать эти усилия и предложить шаги по организации сообщества вокруг последовательной и открытой науки. практики. Это действие переводится в общие методологические рекомендации или предложения, которые мы представляем после типичного рабочего процесса эксперимента smFRET, включая подготовку и определение характеристик образцов, описание установки, сбор и сохранение данных, а также анализ данных.Эти рекомендации о том, как «практиковать» smFRET, являются не попыткой систематизировать сообщество, а скорее первоначальным предложением, которое направлено на поощрение открытого диалога о существующих практиках в нашей области и приводит к более высокой воспроизводимости результатов экспериментов smFRET. Затем мы обсуждаем практику открытой науки, а также первые шаги, которые были предприняты для формирования международного сообщества FRET. В конце мы выделим несколько областей, в которых smFRET окажет большое влияние в различных областях науки в ближайшем будущем.

    Биологический ключ лаборатории определения группы крови

    Система кровообращения. Проследите путь крови через бьющееся сердце и сеть кровеносных сосудов, по которым кровь поступает в тело. Возьмите образцы крови из разных кровеносных сосудов, чтобы наблюдать за кровяными тельцами и измерять уровни кислорода, углекислого газа, сахара и мочевины.

    Mm3 tuner 5.9 cummins

    • Человек с кровью типа А имеет 1 аллель от мамы и один аллель от отца. Когда вы соединяете их вместе, чтобы сформировать потомство во время оплодотворения, может быть два разных генотипа для получения крови типа A: IAIA = кровь типа A IAIO = кровь типа A Человек с кровью типа B имеет 1 аллель от своей мамы и один аллель от их папа.
    • ★★★ Правильный ответ на вопрос: Лабораторная деятельность: Тип крови, родословная, тайна, тайна в Уэксфорде, ключ ответа — edu-answer.com

    Blood Typing — Интерактивная игра, в которой вы можете узнать о группах крови, а также определить, какой тип крови пострадавшему от несчастного случая необходимо переливание крови. Late Nite Labs — включает в себя как биологические, так и химические лаборатории. Объекты обучения TeqSmart: наука — включает интерактивные клетки животных и растений …

    Человеческие черты Лабораторное занятие Носок Кариотипическое задание Лабораторные инструкции для малышей Reebop — также: Советы лаборатории Reebop PPT Макияж для малышей Reebop: генетика с улыбкой Генетика Практика простых скрещиваний ( горох) Практика генетики Скрещивание по двум признакам (кролики) и (морские свинки) Практика генетики по признакам, связанным с полом и группам крови (получите раздаточный материал в классе)

    3.Назовите два сходства между диабетом 1 и 2 типа. Оба связаны с более высоким, чем обычно, уровнем сахара в крови. Оба связаны с неспособностью клеток транспортировать глюкозу из крови в клетки. Оба они приводят к серьезным осложнениям, если их не лечить. Оба могут привести к смерти. Здоровая диета и физические упражнения являются частью лечения обоих … В качестве предварительного просмотра будущего раздела по размножению растений обратите внимание, что из кукурузы образуются два разных типа цветов — один мужской (видно кистями) и один женский (виден шелка).Кукурузный початок на самом деле представляет собой коллекцию из более чем сотни потомков, аккуратно упакованных в початки, способных храниться долгое время и идеально подходящих для изучения генетики.

    Когда кровь течет по кровеносной системе, она проходит через три типа кровеносных сосудов: артерии, капилляры и вены. Раздел 37-2. Кровь и лимфатическая система. Красные кровяные тельца переносят кислород. Лейкоциты атакуют чужеродные вещества или организмы.

    Лаборатория: Использование анализов крови для выявления преступников и младенцев. Часть 1.Группы крови (система ABO) В системе ABO четыре группы крови: тип A, тип B, тип AB и тип O. Эти группы крови относятся к различным версиям молекул углеводов (сложных сахаров), которые присутствуют на поверхности. красных кровяных телец. Люди с: Имеют: кровь типа A

    Icel chant mass pdf

    Все ресурсы (чтение, картинки, PowerPoint, рабочие листы, обзоры, тесты …) доступны сейчас на этом веб-сайте, наслаждайтесь !! Об авторе Бреннон Сапп

    Page 9 AP Biology: 2013 Exam Review Практика мышления 1.Для каждой молекулы, показанной справа, ответьте на следующие вопросы с обоснованием каждой из них: a. Он полярный или неполярный? б. Он гидрофобный или гидрофильный? c. Потребуется ли молекуле протеиновый канал для переноса в клетку? 2.

    Определяются цельная кровь, плазма, сыворотка и различные типы клеток крови. Ключевые понятия Кровь представляет собой сложную жидкость, состоящую из множества различных компонентов, но ее можно разделить на твердые вещества (красные кровяные тельца, лейкоциты и тромбоциты) и жидкость (плазма).

    «Ткани животных»%% четыре% типов%% тканей% найдено%% животных: %% эпителиальная ткань, соединительная ткань, мышечная ткань и «нервная ткань».% В% эта% лаборатория …

    Пока мы говорим о ключе ответов на рабочие листы «Уголок биологии», прокрутите вниз, чтобы увидеть определенные связанные изображения, чтобы дополнить свои идеи. Ключ ответа на концептуальную карту крови, ключ-ответ на концептуальную карту пищеварительной системы и ответы на концептуальную карту нервной системы — вот некоторые основные вещи, которые мы покажем вам на основе названия галереи.

    Часть B — Краткие ответы 1.Мочевина является отходом метаболизма АМИНОКИСЛОТ, вырабатывается ПЕЧЕНЬЮ и выводится ПОЧКАМИ. 2. Основным типом отходов, обнаруживаемых в моче, являются АЗОТНЫЕ отходы. 3. Самая удаленная часть почки называется КОРОКОЙ. Средний слой называется МЕДУЛЛА. Моча собирается в полости, называемой почечными … … Аллели группы крови ABO привели к постоянному потоку запросов в Биологический проект: «Я читал вашу информацию о наследовании групп крови и очень запутался! Сравните свои ответы с доктором.Халлика. Мы рекомендуем ознакомиться с его объяснением …

    Метаболизм глюкозы также связан с риском развития диабета 2 типа. Человеческий организм производит много глюкозы. Этот избыток глюкозы быстро транспортируется к клеткам организма потоком крови и быстро превращается в глюкозу. Когда организм испытывает дефицит энергии, он будет пытаться использовать любую глюкозу, которую он может …

    Тип файла PDF Ответить ключ к лаборатории гомеостаза Макгроу Хилл Ответить ключ к лаборатории гомеостаза Макгроу Хилл Когда люди должны пойти в книжные магазины, начать поиск по магазину, полка за полкой, на самом деле это проблематично.Вот почему мы предлагаем сборники книг на этом сайте. Это значительно облегчит вам поиск ответов на вопросы руководства по лаборатории гомеостаза …

    Usmc brag sheet navy

    Dragon idle game

    • Неизвестная группа крови; если никакой реакции нет, значит, ее нет; O Популярные учебные материалы по биологии 202.

      Студенты определяют группу крови четырех человек. Настоящие медицинские пластины для определения группы крови добавляют реалистичности. После сбора данных учащиеся рассматривают вопросы об отцовстве и генетике четырех испытуемых.Верните этот важный урок биологии в свой класс. Студентам это нравится.

    • Человеческие черты Лабораторная деятельность Носок Кариотипия Лабораторные инструкции Baby Reebop — также: Советы Reebop Lab — PPT Baby Reebop Makeup: Генетика с улыбкой Генетика Практика простых скрещиваний (растения гороха) Генетика Практика Двухпредметные скрещивания (кролики) и Пейдж 2 (морские свинки) Генетика Практикуйте связанные с полом черты и группы крови (получите раздаточный материал в классе)

      Запомните путь кровотока через сердце, прежде чем пытаться это сделать! 5.Опишите взаимосвязь между структурой капилляров и их функцией. Стенки капилляра толщиной в одну клетку состоят из этих плоских клеток. Капилляры — это место распространения кислорода из легких в кровь и из крови в ткани.

    Примеры наглядных прилагательных в словах

    • Группы крови человека 154 Определение группы крови по системе ABO основано на антигенах A и B. Группа крови резус-фактора основана на резус-факторе 156.4 Заболевания крови 155 8,4 157 8,5 Очищение крови магнитами 158

      Виртуальная лаборатория; Рабочий лист клеточного цикла и рака. Ваше тело моложе, чем вы думаете. Глава 10 PA Study Guide 2017-2018. Глава 11: Результаты исследования обезьяны. Типирование крови. Глава 11 Учебное пособие. Глава 11.1 PPT. Глава 11.2 PPT. Глава 11.3 PPT. Глава 11.4 PPT. Глава 11.5 PPT. Результаты исследования Trait Lab 2016. Генетика Bikini Bottom …

    Молекулярная орбитальная диаграмма h3 Не удалось запустить сервисный блок mysqld Сервис mysqld не найден redhat

    • Аксессуары Nuwave brio 14q

    • Greatsword of artorias2a 2005

      dodge ram 2500, тяговое усилие 5-го колеса

    • Код ошибки Dism 87 windows 7

    • Частота сканера полиции округа Кук

    • Формула расчета стоимости плазменной резки в Excel

    • Угол атаки Naca 2412

    • Какие тесты на наркотики использует bridgestone?

    • Учетная запись Reliacard в Usbank заблокирована

    • Калькулятор траектории мяча для гольфа excel

    • Gmc envoy теряет мощность во время вождения

      Виниловые наклейки для плитки для ванной комнаты на кухне

    • Часовой пояс клиента Spring загрузки 900 03
    • Бензиновые двигатели Zenoah uk

      Ucps directory

    • Panzer bp12 встречает promag mka a1

      Линейная алгебра и ее приложения Руководство по решениям 5-го издания pdf

    • Bmw f30 wide body kit

    • Тест на здоровье абдоминальных теней Тины Джонс

      План выпуска Servicenow Stream

    • Аксессуары для коронок Fx

      Обновление Asus rog g20aj

    • Лампа из эпоксидного дерева diy

      Swappa imei

      0003

    • Согласно автору поведение лидерства во взаимоотношениях сводится к следующему, за исключением

      Google сообщения функции чата не работают

    • Калькулятор источника питания лампового усилителя

      Hog slammer band

    • Аренда места проведения kumeu

      Twitch ke eps замирает, но звук продолжается

    • Ножки для замены Razer deathadder chroma

      Объективы Canon RF

    Chevelle 1970 на продажу Craigslist arizona Цена на программное обеспечение Tajima

    Стоимость мобильного телефона Mi y2 в Индии сегодняDmt музыкальный фестиваль

    34 9033 Электрогитара Scott
    J.п. стажировка инженера-программиста morgan

    Pedersoli Cook Underhammer

    Turtle beach elite 800x rx

    Дробовик Churchill 612

    Ткань, физиология, уровень организации в многоклеточных организмах он состоит из группы структурно и функционально подобных клеток и их межклеточного материала. По определению, у одноклеточных организмов ткани отсутствуют. Узнайте больше о тканях в этой статье.27 ноября 2020 г. · То есть положительная группа крови может фактически передавать положительную или отрицательную группу крови. Однако отрицательная группа крови может передаваться только отрицательной. Таким образом, человек с группой крови B + может передавать кровь B или O, как положительную, так и отрицательную. Дейтон, 9 июля 2007 г. Привет, глаза! Вы правильно спросили — это невозможно. Как соединить точки в графике matlab
    Список форумов по оружию

    Курсор цветности

    Вам сдается одна карта из колоды из 52 карт.найдите вероятность того, что вам не выдадут 6.

    Абзац для нее, когда она на вас злится

    Это действительно имеет отношение к ключу ответа онлайн-активности для определения группы крови. Как только вы перейдете в Yahoo Answers, вы действительно сможете отправить свою проблему с построением обратной ссылки. Одновременно вы можете ответить на несколько запросов, чтобы решить проблему с построением ссылок, учитывая, что каждый раз, когда вы решаете проблему Yahoo, вы будете получать гиперссылку.кровь ко всем другим группам крови. Используя свои знания о группах крови и иммунной системе, объясните, почему это правда. Группа крови O вообще не содержит аутоантигенов, поэтому, когда она смешана с другой группой крови, она не стимулирует выработку антител в другой группе крови и, таким образом, считается неопасным чужеродным веществом. Кошелек Colibra
    Слова, которые означают знакомство с

    Разъем кабеля для вывоза мусора Moen

    Топливный насос Vanguard, 16 л.с. извинения.Произошла ошибка при установке вашего пользовательского файла cookie. Чтобы продолжить, настройте свой браузер на прием файлов cookie. NEJM.org использует файлы cookie для повышения производительности за счет запоминания идентификатора сеанса при переходе со страницы на страницу. Этот файл cookie хранит только идентификатор сеанса; нет другой информации … Что такое хорошее сообщение об отсутствии на работе на праздники
    Firefox Theme Creator

    Dell optiplex 7070 загрузка через usb

    шахматы с двойной атакой

    Daniel ehrhardt

    Плазмолиз «Мокрая лаборатория» Исследовательская деятельность: Осмос в клетках элодеи или луковых клетках Введение. Одна из функций клеточной мембраны — контролировать поток материалов в клетку и из нее. Наклейка Glock mag
    Beech Staggerwing Club

    Kenwood dnx9990hd не включается

    Перегиб на графике

    Формула скользящей шкалы

    903 Анализ различных образцов ДНК при расследовании изнасилования показан на этом рисунке. Образцы ДНК загружали в следующие дорожки: 1. Известный образец крови жертвы. 2. известный образец крови ответчика.3. Маркеры размера ДНК. 4. Женская фракция из влагалищного мазка жертвы. Домашнее задание: Лаборатория 8 КЛЮЧ ОТ ОТВЕТА 3. Х-сцепленные черты Облысение — это Х-сцепленная черта у людей. Мужчины, унаследовавшие Х-хромосому с аллелем облысения, будут … Определение группы крови …

    Дата контракта с УСПС Кому принадлежит мемориал достоинства

    Среднее время диалога Графана за время

    Как получить панцирь призрака гарпии

    D
    Бинокль Swift

    Hpe vmware esxi 6.5 u3 загрузить

    Cushman trackter cab

    19 марта 2016 г. · Лаборатория виртуальной лаборатории идентификации бактерий. Цель лаборатории — познакомить вас с наукой и методами, используемыми для идентификации различных типов бактерий на основе их последовательность ДНК. Серия виртуальных лабораторий hhmis биоинтерактивная идентификация бактерий эволюция колюшки эволюция трансгенных мух иммунология кардиология нейрофизиология подробнее … pdecandia.com | home
    Is palmerpercent27s vegan
    Texas socks5 список доверенных лиц

    Secret Church 10

    Salary учитель
    3

    Коробка для заката x8

    Подсчет клеток крови указывает на наличие или отсутствие заболевания крови.Выполняются тесты, которые можно использовать для подсчета эритроцитов, лейкоцитов, и количество может быть сопоставлено с нормальным подсчетом, как показано в ключе лаборатории. Кроме того, можно также отметить и сравнить форму и внешний вид и … Лаборатория определения группы крови — SlideShare. Slideshare.net Лаборатория по определению группы крови 1. Лаборатория по определению группы крови Цели Цели — узнать, как определить группу крови и найти подходящего донора для пациента. Материалы Четыре образца крови Сыворотка анти-А Сыворотка анти-В Сыворотка анти-резус-фактора 4 Одноразовые микропипетки 4 Слайды для типирования крови 12 палочек для смешивания Контейнер для биологической опасностиMethodFist…
    Tikz label on line
    Vpn india free

    Скачать песни из Spotify в mp3

    John deere 455 curtis cab
    Как найти произведение трех чисел

    Как исправить отставание зеркального отображения экрана samsung tv

    Btc miner pro
    Центр заключения для несовершеннолетних на реке Гила

    Персонализированные чашки йети с изображениями

    Винтажные гитары bc rich на продажу
    6
    Dianna 9000ini
    1800 кухонных плит
    Что означает статус платежа невыполненный означает edd

    Winchester white box 9mm 147 gran jhp review

    Используйте таблицу значений для оценки выражений ниже
    Будет ли снова падение запасов

    Remi apparels ltd

    Бесплатные практические тесты и другие • ресурсы для тестирования, разделенные на 300 категорий, включая академические, карьерные, личностные, интеллектуальные и другие.Руководство по выставлению оценок AP® Biology 2009. Совет колледжа. Совет колледжей — это некоммерческая членская ассоциация, миссия которой состоит в том, чтобы помочь студентам получить доступ к успехам и возможностям колледжа. Основанная в 1900 году ассоциация состоит из более чем 5600 школ, колледжей, университетов и других образовательных организаций.

    Эртугрул, сезон 1, серия 45, английские субтитры dailymotionKakegurui x fem reader, лимон

    Inttilun
    Indiana Craigslist cars
    Diy slime kit поставляет
    1920×1080 vs 2560×1440

    Бульдозер для дерева

    Пакет Eurovan baja

    Энди Аджлуни чистая стоимость

    С 1994 года CELLS живы! предоставил студентам учебные ресурсы по клеточной биологии, микробиологии, иммунологии и микроскопии с помощью удобных для мобильных устройств интерактивных анимаций, видео, головоломок, викторин и учебных пособий.

    Jenkins job dsl generic triggerHonda magna v45 bobber kit

    limited
    Запуск сценария PowerShell от имени администратора при запуске
    Picrew two characters
    Как закончить нижнюю часть гипсокартона в гараже

    Minecraft _ плавающий ковер

    Chrome windows 7 32-разрядная загрузка в автономном режиме
    2

    Советы и хитрости игры House flip

    Federal hst sig ballistics

    AQA GCSE Biology Экзаменационные документы GCSE и макеты GCSE 2021



    Наши ресурсы практики GCSE

    Во-первых, наши практические тесты GCSE.Затем, во-вторых, наша редакция GCSE 2021 года. В-третьих, наши новые тестовые экзамены GCSE.

    Плюс, наш выпускной экзамен на 2021 год — насмешки. А также наши практические тесты GCSE по математике. Вы также можете найти полезными наши практические тесты по математике для GCSE и наши 12+ GCSE.

    Не забудьте также посетить страницы с практикой сдачи экзаменов GCSE по английскому и другим языкам и Past Papers по гуманитарным наукам.

    Биологические документы GCSE

    Во-первых, прошлые GCSE экзаменационной комиссии AQA

    AQA-GCSE Biology Paper 1F Specimen (2018)

    AQA-GCSE Biology Paper 1F Specimen (2018) 9000 Mark Scheme

    -GCSE Biology Paper 1H Specimen (2018)

    AQA- GCSE Biology Paper 1H Specimes (2018) Mark Scheme

    AQA-GCSE Biology Paper 2F Specimen (2018)

    AQA-GCSE Biology Paper 2F Specimen (2018) Mark Scheme

    AQA-GCSE Biology Paper 2H Образец (2018)

    AQA-GCSE Biology Paper 2H Образец (2018) Mark Scheme



    Наш лучший совет — — 9069 Carry On ‘

    Если ваш ребенок беспокоится о сдаче экзамена, мы советуем заверить его, что с ним все будет в порядке, и просто постарайтесь.Не оставляйте никаких проблем с тревогой, таких как описанные выше, чтобы выздороветь самостоятельно, без каких-либо изобретений … так как это может привести к ситуации, в которой ваш ребенок вообще не сможет сосредоточиться на своей практике.

    Удачи с экзаменами!

    Попробуйте нашу практику GCSE

    Biology GCSE 2017

    AQA-GCSE Biology Foundation B1 (июнь 2017 г.)

    AQA-GCSE Biology Foundation B1 (июнь 2017 г.) Схема оценок

    AQA-GCSE Biology Higher B1 (июнь 2017 г. )

    Biology Higher B1 (июнь 2017 г.) Схема оценок

    AQA-GCSE Biology Foundation B2 (июнь 2017 г.)

    AQA-GCSE Biology Foundation B2 (июнь 2017 г.) Схема оценок

    AQA-GCSE Biology Higher B2 (июнь 2017 г.)

    AQA-GCSE Biology Higher B2 (июнь 2017 г.) Схема оценок

    AQA-GCSE Biology Foundation B3 (июнь 2017 г.)

    AQA-GCSE Biology Foundation B3 (июнь 2017 г.) Схема оценок

    AQA-GCSE Biology Higher B3 (июнь 2017)

    AQA- GCSE Biology Higher B3 (июнь 2017) Схема оценок

    Попробуйте нашу практику GCSE

    Экзаменационные работы AQA Biology GCSE



    Biology GCSE 2016

    AQA-GCSE Foundation (июнь2016)

    AQA-GCSE Biology Foundation B1 (июнь 2016) Схема оценок

    AQA-GCSE Biology Higher B1 (июнь 2016)

    AQA-GCSE Biology Higher B1 (июнь 2016) Схема оценок

    AQA-GCSE Biology Foundation B2 (Июнь 2016)

    AQA-GCSE Biology Higher B2 (июнь 2016)

    AQA-GCSE Biology Higher B2 (июнь 2016) Схема оценок

    AQA-GCSE Biology Foundation B3 (июнь 2016)

    AQA-GCSE Biology Foundation B3 (Июнь 2016 г.) Схема оценок

    AQA-GCSE Biology Higher B3 (июнь 2016 г.)

    AQA-GCSE Biology Higher B3 (июнь 2016 г.) Схема оценок



    Биология GCSE 2015

    Фонд AQA- GCSE B1 (июнь 2015 г.)

    AQA-GCSE Biology Foundation B1 (июнь 2015 г.) Схема оценок

    AQA-GCSE Biology Higher B1 (июнь 2015 г.)

    AQA-GCSE Biology Higher B1 (июнь 2015 г.) Схема оценок

    AQA- GCSE Фонд биологии B2 (июнь 2015 г.)

    AQA- GCSE Biology Foundation B2 (июнь 2015 г.) Схема оценок

    AQA-GCSE Biology Higher B2 (июнь 2015 г.)

    AQA-GCSE Biology Higher B2 (июнь 2015 г.) Схема оценок

    AQA-GCSE Biology Foundation B3 (июнь 2015 г.)

    AQA-GCSE Biology Foundation B3 (июнь 2015 г.) Схема оценок

    AQA-GCSE Biology Higher B3 (июнь 2015 г.)

    AQA-GCSE Biology Higher B3 (июнь 2015 г.) Схема оценок

    Экзаменационные работы AQA Biology GCSE

    Химия Экзаменационные работы GCSE

    Во-первых, прошлые экзаменационные работы от экзаменационной комиссии AQA

    ПЕРИОДИЧЕСКАЯ ТАБЛИЦА

    AQA-GCSE Chemistry Paper 1F (Образец 2018)

    AQA-GCSE Chemistry Paper 1F (Образец 2018) Схема оценок

    AQA-AQA Химическая бумага 1H (образец 2018)

    AQA-GCSE Химическая бумага 1H (образец 2018) Схема отметок

    Химическая бумага AQA-GCSE 2F (Образец 2018)

    Химическая бумага AQA-GCSE 2F (Образец 2018) Схема маркировки

    -GCSE Химический документ 2H (образец 2018)

    AQA-GCSE Chemistry Paper 2H (Образец 2018) Схема оценок

    Химический документ AQA-GCSE 1F (июнь 2018)

    AQA-GCSE Chemistry Paper 1F (июнь 2018) Схема оценок

    AQA -GCSE Chemistry Paper 1H (июнь 2018)

    AQA- GCSE Chemistry Paper 1H (июнь 2018) Mark Scheme

    AQA-GCSE Chemistry Paper 2F (июнь 2018)

    AQA-GCSE Chemistry Paper 2F (июнь 2018) Схема оценок

    AQA-GCSE Chemistry Paper 2H (июнь 2018)

    AQA-GCSE Chemistry Paper 2H (июнь 2018) Mark Scheme

    Попробуйте нашу практику GCSE

    More Chemistry GCSE 2017

    AQA-GCSE Chemistry INSERT 1F и 1H (июнь 2017 г. )

    AQA-GCSE Chemistry Paper 1F (июнь 2017)

    AQA-GCSE Chemistry Paper 1F (июнь 2017) Mark Scheme

    AQA-GCSE Chemistry Paper 1H (июнь 2017)

    AQA-GCSE Chemistry Paper 1H (июнь 2017 г.) ) Отметьте схему

    AQA- GCSE Chemistry INSERT 2F и 2H (Июнь 2017)

    AQA-GCSE Chemistry Paper 2F (июнь 2017)

    AQA-GCSE Chemistry Paper 2F (июнь 2017) Mark Scheme

    AQA-GCSE Chemistry Paper 2H (июнь 2017)

    AQA-GCSE Chemistry Paper 2H (Июнь 2017) Mark Scheme

    AQA-GCSE Chemistry INSERT 3F и 3H (июнь 2017)

    AQA-GCSE Chemistry Paper 3F (июнь 2017)

    AQA-GCSE Chemistry Paper 3F (июнь 2017) Mark Scheme

    AQA- GCSE Chemistry Paper 3H (июнь 2017)

    AQA-GCSE Chemistry Paper 3H (июнь 2017) Mark Scheme

    AQA Biology экзаменационные работы GCSE

    Попробуйте нашу практику GCSE

    Chemistry 2016

    Сначала от экзаменационной комиссии AQA.

    AQA-GCSE Chemistry INSERT 1F и 1H (июнь 2016)

    AQA-GCSE Chemistry Paper 1F (июнь 2016)

    AQA-GCSE Chemistry Paper 1F (июнь 2016) Схема оценок

    AQA-GCSE Chemistry Paper 1H (июнь 2016)

    AQA-GCSE Chemistry Paper 1H (июнь 2016) Mark Scheme

    AQA-GCSE Chemistry INSERT 2F и 2H (июнь 2016)

    AQA-GCSE Chemistry Paper 2F (июнь 2016)

    AQA-GCSE Chemistry Paper 2F (Июнь 2016) Mark Scheme

    AQA-GCSE Chemistry Paper 2H (июнь 2016)

    AQA-GCSE Chemistry Paper 2H (июнь 2016) Mark Scheme

    AQA-GCSE Chemistry Paper 3F (июнь 2016)

    AQA-GCSE Chemistry Документ 3F (июнь 2016 г.) Схема оценок

    AQA-GCSE Chemistry Paper 3H (июнь 2016 г.)

    AQA-GCSE Chemistry Paper 3H (июнь 2016 г.) Схема оценок

    Попробуйте нашу практику GCSE

    Chemistry 2015

    Во-первых, из Экзаменационная комиссия AQA

    Документ по химии AQA-GCSE 1F (июнь 2015 г.)

    AQA-GCSE Chemistry Paper 1F (июнь 2015) Mark Scheme

    AQA-GCSE Chemistry Paper 1H (июнь 2015)

    AQA-GCSE Chemistry Paper 1H (июнь 2015) Mark Scheme

    AQA-GCSE Chemistry Paper 2F (июнь 2015 г. )

    AQA-GCSE Chemistry Paper 2F (июнь 2015 г.) Mark Scheme

    AQA-GCSE Chemistry Paper 2H (июнь 2015)

    AQA-GCSE Chemistry Paper 2H (июнь 2015 г.) Схема оценок

    AQA-GCSE Chemistry Paper 3F ( Июнь 2015)

    AQA-GCSE Chemistry Paper 3F (июнь 2015) Mark Scheme

    AQA-GCSE Chemistry Paper 3H (июнь 2015)

    AQA-GCSE Chemistry Paper 3H (июнь 2015) Схема оценок

    Physics 13+ экзамен документы

    Physics (Haberdashers ‘Aske’s 2015)
    13+ Physics (ISEB Level 2 2017)
    13 + Physics Mark Scheme (ISEB Level 2 2017)

    Physics (Merchant Taylors 2017)

    AQA Biology экзаменационные работы

    Попробуйте нашу практику GCSE

    Наука 13+ экзаменационных работ

    Наука (Школа Свободных людей Лондона)
    Наука 13+ (Школа Свободных людей Лондона, 2014 г.)
    Наука (Колледж Далвич)
    Наука 13+ (Школа Эмануэля)
    Наука (Уровень 1 ISEB, 2017 г.) .
    Схема научных отметок 13+ (ISEB, уровень 1 2017)
    Наука (Стипендия Святого Эдварда, 2016 г.)
    13+ Наука (Стипендия Святого Эдуарда, 2015 г.)
    Наука (Стипендия Святого Эдуарда, 2016 г.)
    Наука 13+ (Стипендия Святого Эдуарда, 2014 г.).
    Наука (St Edward’s 2013)
    13+ Science (St Edward’s 2013-14)
    Science (St Edward’s 2013)
    13+ Science (Стипендия Святого Эдварда 2011)

    Наконец, 13+ Science (Whitgift).

    Экзаменационные работы AQA по биологии GCSE

    14 + Экзамены по науке

    Во-первых, 14+ Естествознание (Лондонская школа Freemen’s School 2014)

    Во-вторых, 14+ Science (St Edward’s 2016)

    В-третьих, 14+ Science (St Edward’s 2014)

    Наконец, 14+ Science (St Edward’s 2013).

    Попробуйте нашу практику GCSE

    Как преуспеть в тестах на числовое мышление

    Общий совет, который дается, типичен для того, чтобы некоторая целенаправленная практика тестирования улучшила ваш результат. К сожалению, хорошие математические способности не приносят быстрых результатов.

    • Дайте свой «первый ответ».
    • В качестве теста с ограничением по времени вам нужно в среднем около одной минуты на каждый вопрос. Работайте бодро, но аккуратно.
    • Каждый вопрос считается одинаковым, поэтому сначала выбирайте простые и не тратьте время теста на самые сложные вопросы.
    • Практикуйте некоторые из наиболее распространенных типов числовых тестов на основных веб-сайтах издателей тестирования. Только по этой причине мы настоятельно рекомендуем практиковать образцы вопросов с сайтов Kenexa-IBM, TalentQ и SHL. убедитесь, что вам удобно пользоваться таблицами данных, интерпретировать графики и манипулировать большими финансовыми цифрами.

    Как хорошо сдать тесты на вербальное мышление

    Они бывают разных форматов. Традиционный формат понимания состоит в том, чтобы иметь короткий текстовый отрывок, за которым следует серия вопросов о фактах, мнениях, выводах из содержания отрывка.Немного похоже на те тесты по английскому в начальной школе, где вы отвечали на вопросы по отрывку из романа. Независимо от типа теста, важно помнить:

    • Внимательно прочитать каждый вопрос. Часто вопросы основаны на 1-2 ключевых словах, поэтому вы должны более внимательно их интерпретировать.
    • Если спросить, применимо ли что-то «всегда», в то время как отрывок утверждает, что это «иногда» имеет место, то это ложное толкование.
    • Одна из полезных стратегий — сначала просмотреть отрывок, а затем прочитать его более подробно.
    • Ответить на каждый вопрос будет эффективнее, если вы приблизительно вспомните, где найти ответ в отрывке.

    Как преуспеть в тестах абстрактного мышления

    Тесты абстрактного мышления просят вас искать изменяющиеся шаблоны на «картинках». Более простые вопросы, обычно в начале теста, включают одно изменение цвета, положения, размера и т. Д. Показанных фигур.

    Вопросы становятся более сложными, так как вы должны заметить два или три изменения в любой из показанных функций.

    Author: alexxlab

    Добавить комментарий

    Ваш адрес email не будет опубликован. Обязательные поля помечены *